PN 131 Comprehensive Final NCLEX Practice

¡Supera tus tareas y exámenes ahora con Quizwiz!

Which statement indicates that an expectant mother understands the diagnosis of placenta previa? "My doctor will not let my pregnancy go beyond my due date before he induces me." "My doctor will monitor for rupture of membranes each week at my appointment." "My doctor will not induce labor at any time during this pregnancy." "My baby will probably come early because of my condition."

"My doctor will not induce labor at any time during this pregnancy." The physician will not induce a patient with this diagnosis. Rupture of membranes is not a primary risk for this complication. This diagnosis does not have a high correlation with preterm births

Signs of ICP in a 3 year old child include: 1. headache, lethargy 2. high pitched cry, bulging fontanels 3. apnea, crossed eyes 4. painful head movement, anorexia

1 Subtle signs of increased ICP in a 3-year-old child include a change in alertness or muscle twitching. Headache is also an indicator of ICP.

The nurse is assessing the reflexes of a newborn infant. In eliciting the Moro reflex, the nurse should perform which action? 1.Make a loud, abrupt noise to startle the newborn. 2.Stimulate the ball of the foot of the newborn by firm pressure. 3.Stimulate the perioral cavity of the newborn infant with a finger. 4. Stimulate the pads of the newborn infant's hands by firm pressure.

1 The Moro reflex is elicited by placing the newborn on a flat surface and striking the surface or making a loud, abrupt noise to startle the newborn. The newborn assumes sharp extension and abduction of the arms with the thumbs and forefingers in a C position; this is followed by flexion and adduction to an "embrace" position (legs follow a similar pattern). The Moro reflex is present at birth and is absent by 6 months of age if neurological maturation is not delayed. A persistent response lasting more than 6 months may indicate a neurological abnormality. The rooting reflex is elicited by stimulating the perioral area with the finger. The palmar grasp reflex is elicited by stimulating the palm of the hand by firm pressure, and the plantar grasp reflex is elicited by stimulating the ball of the foot by firm pressure.

A nurse is teaching parents about accident prevention for a toddler. Which guidelines is most appropriate? Select all that apply 1. Always make the toddler wear a seat belt when riding in a car. 2. Make sure all medications are kept in containers with childproof safety caps. 3. Never leave a toddler unattended on a bed. 4. Teach rules of the road for bicycle safety.

1, 2 RATIONALE: Making sure all medications are kept in containers with childproof safety caps is the most appropriate guideline because poisoning accidents are common in toddlers owing to the toddler's curiosity and his increasing mobility and ability to climb. When riding in a car, a toddler should be strapped into a car seat, not a seat belt. A seat belt is an appropriate guideline for a school-age child. Never leaving a child alone on a bed is an appropriate guideline for parents of infants. Toddlers already have the ability to climb on and off of beds and other furniture by themselves. Note, however, that toddlers should never be left unattended on high surfaces, such as an examining table in a physician's office. Teaching the rules of the road for bicycle safety is an appropriate safety measure for a school-age child. Toddlers shouldn't be allowed in the road unsupervised.

A nurse is collecting data on an infant with a diagnosis of suspected Hirschsprung's disease. Which of the following questions to the mother will most specifically elicit information regarding this disorder? 1. "Does your infant have foul-smelling, ribbon-like stools?" 2. "Is your infant constantly vomiting?" 3. "Does your infant constantly spit up feedings?" 4. "Does your infant have diarrhea?"

1. "Does your infant have foul-smelling, ribbon-like stools?" Rationale: Chronic constipation, beginning in the first month of life and resulting in pellet-like or ribbon-like stools that are foul smelling, is a clinical manifestation of Hirschsprung's disease. Delayed passage or absence of meconium stool in the neonatal period is the cardinal sign. Bowel obstruction, especially in the neonatal period, abdominal pain and distention, and failure to thrive are also clinical manifestations. Options 2, 3, and 4 are not specific clinical manifestations of this disorder.

The nurse is providing postpartum instructions to a client who will be breast-feeding her newborn. The nurse determines that the client has understood the instructions if she makes which statements? Select all the apply. 1. "I should wear a bra that provides support." 2. "Drinking alcohol can affect my milk supply." 3. "The use of caffeine can decrease my milk supply." 4. "I will start my estrogen birth control pills again as soon as I get home." 5. "I know if my breasts get engorged I will limit my breast feeding and supplement the baby." 6. "I plan on having bottled water available in the refrigerator so I can get additional fluids easily."

1. "I should wear a bra that provides support." 2. "Drinking alcohol can affect my milk supply." 3. "The use of caffeine can decrease my milk supply." 6. "I plan on having bottled water available in the refrigerator so I can get additional fluids easily."

A nurse is teaching a parent how to administer antibiotics at home to a toddler with acute otitis media. Which statement by the parent indicates that teaching has been successful? 1. "I'll give the antibiotics for the full 10-day course of treatment." 2. "I'll give the antibiotics until my child's ear pain is gone." 3. "Whenever my child is cranky or pulls on an ear, I'll give a dose of antibiotics." 4. "If the ear pain is gone, there's no need to see the physician for another examination of the ears."

1. "I'll give the antibiotics for the full 10-day course of treatment." RATIONALE: The mother demonstrates understanding of antibiotic therapy by stating she'll give the full 10-day course of treatment. Antibiotics must be given for the full course of therapy, even if the child feels well. Otherwise, the infection won't be eradicated. Antibiotics should be taken at ordered intervals to maintain blood levels and not as needed for pain. A reexamination at the end of the course of antibiotics is necessary to confirm that the infection is resolved.

A nurse is evaluating the parent's understanding of discharge care regarding the functioning of the infant's ventricular peritoneal shunt. Which statement by a parent indicates an understanding of the shunt complications? 1. "If my baby has a high-pitched cry, I should call the doctor." 2. "I should position my baby on the side with the shunt when sleeping." 3. "My baby will pass urine more often now that the shunt is in place." 4. "I should call my doctor if my baby refuses purees."

1. "If my baby has a high-pitched cry, I should call the doctor." Rationale: If the shunt is broken or malfunctioning, the fluid from the ventricle part of the brain will not be diverted to the peritoneal cavity. The cerebrospinal fluid will build up in the cranial area. The result is intracranial pressure, which then causes a high-pitched cry in the infant. The baby should not have pressure when on the shunt side. Skin breakdown and possible compression to the apparatus could result. This type of shunt affects the gastrointestinal system, not the genitourinary system. Option 4 is only a concern if the baby becomes malnourished or dehydrated, which could then raise the body temperature. Otherwise, refusal to eat purees has no direct relationship to the shunt functioning.

Parents of a child with cystic fibrosis ask the nurse why their child must receive supplemental pancreatic enzymes. Which response by the nurse is most appropriate? 1. "Pancreatic enzymes promote absorption of nutrients and fat." 2. "Pancreatic enzymes promote adequate rest." 3. "Pancreatic enzymes prevent intestinal mucus accumulation." 4. "Pancreatic enzymes help prevent meconium ileus."

1. "Pancreatic enzymes promote absorption of nutrients and fat." RATIONALE: Pancreatic enzymes are given to a child with cystic fibrosis to aid fat and protein digestion. They don't promote rest or prevent mucus accumulation or meconium ileus.

A mother asks the nurse why her 12-month-old baby gets otitis media more frequently than her 10-year-old son. What should the nurse tell her? 1. "The baby's eustachian tubes are shorter and lie more horizontally." 2. "The baby is too young to blow his nose when he has a cold." 3. "The baby spends more time lying down than his older brother; therefore, more dirt gets in the baby's ear." 4. "The baby puts dirty toys in his mouth."

1. "The baby's eustachian tubes are shorter and lie more horizontally." RATIONALE: Infants and young children are more prone to otitis media because their eustachian tubes are shorter and lie more horizontally. Pathogens from the nasopharynx can more readily enter the eustachian tube of the middle ear. The inability to clear nasal passages by blowing the nose, lying down on the floor, and putting dirty toys in the mouth don't increase the tendency toward otitis media.

A child with iron deficiency anemia is ordered ferrous sulfate (Ferralyn), an oral iron supplement. When teaching the child and parent how to administer this preparation, the mother asks why she needs to mix the supplement with citrus juice. Which response by the nurse is best? 1. "The vitamin C in the citrus juice helps with iron absorption." 2. "Having food and juice in the stomach helps with iron absorption." 3. "The citrus juice counteracts the unpleasant taste of the iron." 4. "There isn't a specific reason for it."

1. "The vitamin C in the citrus juice helps with iron absorption." RATIONALE: Administering an oral iron supplement such as ferrous sulfate with citrus juice or another vitamin C source enhances its absorption. Preferably, doses should be administered between meals because gastric acidity and absence of food promote iron absorption. Although citrus juice may improve the taste of an oral iron supplement, this isn't the primary reason for mixing the two together. Telling the mother that there isn't a specific reason for mixing the supplement with citrus juice is inappropriate and inaccurate.

A toddler develops acute otitis media and is ordered cefpodoxime proxetil (Vantin) 5 mg/kg P.O. every 12 hours. If the child weighs 22 lb (10 kg), how many milligrams will the nurse administer with each dose? 1. 50 mg 2. 100 mg 3. 110 mg 4. 220 mg

1. 50 mg RATIONALE: The dose is 5 mg/kg and the child weighs 10 kg. To determine the dose, the nurse would calculate: 5 mg/1 kg × 10 kg = 50 mg per dose.

The nurse in the labor room is caring for a client in the active stage of the first phase of labor. The nurse is assessing the fetal patterns and notes a late deceleration on the monitor strip. What is the most appropriate nursing action? 1. Administer oxygen via face mask. 2. Place the mother in a supine position. 3. Increase the rate of the oxytocin IV infusion 4. Document the findings and continue to monitor the fetal patterns.

1. Administer oxygen via face mask. Late decelerations are due to uteroplacental insufficiency and occur because of decreased blood flow and oxygen to the fetus during uterine contractions. Hypoxemia results; administer 8-10 L/minute via face mask.

A nurse obtains a health history from a mother of a 15-month-old child before administering a measles, mumps, and rubella (MMR) vaccine. Which of the following is essential information to obtain before the administration of this vaccine? 1. Allergy to eggs 2. A recent cold 3. The presence of diarrhea 4. Any recent ear infections

1. Allergy to eggs Rationale: Before the administration of MMR vaccine, a thorough health history needs to be obtained. MMR is used with caution in a child with a history of allergy to gelatin or eggs because the live measles vaccine is produced by chick embryo cell culture. MMR also contains a small amount of the antibiotic neomycin. Options 2, 3, and 4 are not contraindications to administering this immunization.

Which action should a nurse include in the care plan for a 2-month-old infant with heart failure? 1. Allow the infant to rest before feeding. 2. Bathe the infant and administer medications before feeding. 3. Weigh and bathe the infant before feeding. 4. Feed the infant when he cries.

1. Allow the infant to rest before feeding. RATIONALE: Because feeding requires so much energy, an infant with heart failure should rest before feeding. Bathing and weighing the infant and administering medications should be scheduled around feedings. An infant expends energy when crying

Which action should a nurse include in the care plan for a 2-month-old infant with heart failure? 1. Allow the infant to rest before feeding. 2. Bathe the infant and administer medications before feeding. 3. Weigh and bathe the infant before feeding. 4. Feed the infant when he cries.

1. Allow the infant to rest before feeding. RATIONALE: Because feeding requires so much energy, an infant with heart failure should rest before feeding. Bathing and weighing the infant and administering medications should be scheduled around feedings. An infant expends energy when crying; therefore, it's best if the infant doesn't cry.

A toddler with a ventricular septal defect is receiving digoxin (Lanoxin) to treat heart failure. Which assessment finding should be the nurse's priority concern? 1. Bradycardia 2. Tachycardia 3. Hypertension 4. Hyperactivity

1. Bradycardia RATIONALE: Digoxin enhances cardiac efficiency by increasing the force of contraction and decreasing the heart rate. An early sign of digoxin toxicity is bradycardia (an abnormally slow heart rate). To help detect digoxin toxicity, the nurse always should measure the apical heart rate before administering each digoxin dose. Other signs and symptoms of digoxin toxicity include arrhythmias, vomiting, hypotension, fatigue, drowsiness, and visual halos around objects. Tachycardia, hypertension, and hyperactivity aren't associated with digoxin toxicity.

A toddler with a ventricular septal defect is receiving digoxin to treat heart failure. Which assessment finding should be the nurse's priority concern? 1. Bradycardia 2. Tachycardia 3. Hypertension 4. Hyperactivity

1. Bradycardia RATIONALE: Digoxin enhances cardiac efficiency by increasing the force of contraction and decreasing the heart rate. An early sign of digoxin toxicity is bradycardia (an abnormally slow heart rate). To help detect digoxin toxicity, the nurse always should measure the apical heart rate before administering each digoxin dose. Other signs and symptoms of digoxin toxicity include arrhythmias, vomiting, hypotension, fatigue, drowsiness, and visual halos around objects. Tachycardia, hypertension, and hyperactivity aren't associated with digoxin toxicity.

A nurse is caring for a 17-year-old girl with cystic fibrosis who has been admitted to the hospital to receive antibiotics and respiratory treatment for exacerbation of a lung infection. The girl has a number of questions about her future and the consequences of the disease. Which statements about the course of cystic fibrosis are true? Select all that apply. 1. Breast development is delayed. 2. The client is at risk for developing diabetes. 3. Pregnancy and child-bearing aren't affected. 4. Normal sexual relationships can be expected. 5. Only males carry the gene for the disease. 6. By age 20, the client should be able to decrease the frequency of respiratory treatment.

1. Breast development is delayed. 2. The client is at risk for developing diabetes. 4. Normal sexual relationships can be expected. RATIONALE: Cystic fibrosis delays growth and the onset of puberty. Children with cystic fibrosis tend to be smaller than average size and develop secondary sex characteristics later in life. In addition, clients with cystic fibrosis are at risk for developing diabetes mellitus because the pancreatic duct becomes obstructed as pancreatic tissues are destroyed. Clients with cystic fibrosis can expect to have normal sexual relationships, but fertility becomes difficult because thick secretions obstruct the cervix and block sperm entry. Males and females carry the gene for cystic fibrosis. Pulmonary disease commonly progresses as the client ages, requiring additional respiratory treatment — not less.

An infant boy has just had surgery to repair his cleft lip. Which nursing intervention is important during the immediate postoperative period? 1. Cleaning the suture line carefully with a sterile solution after every feeding 2. Laying the infant on his abdomen to help drain fluids from his mouth 3. Allowing the infant to cry to promote lung reexpansion 4. Giving the baby a pacifier to suck for comfort

1. Cleaning the suture line carefully with a sterile solution after every feeding RATIONALE: To avoid an infection that could adversely affect the cosmetic outcome of the repair, the suture line must be cleaned very gently with a sterile solution after each feeding. Laying an infant on his abdomen after a cleft lip repair isn't appropriate because doing so will put pressure on the suture line, causing damage. The infant can be positioned on his side to drain saliva without affecting the suture line. Crying puts tension on the suture line and should be avoided by anticipating the baby's needs, such as holding and cuddling him. Hard objects such as pacifiers should be kept away from the suture line because they can cause damage.

A nurse is caring for an infant with congenital heart disease. Which of the following signs, if noted in the infant, would alert the nurse to the early development of congestive heart failure (CHF)? 1. Diaphoresis during feeding 2. Slow and shallow breathing 3. Pallor 4. Strong sucking reflex

1. Diaphoresis during feeding Rationale: The early symptoms of CHF include tachypnea, poor feeding, and diaphoresis during feeding. Tachycardia would occur during feeding. Pallor may be noted in the infant with CHF, but it is not an early symptom. A strong sucking reflex is unrelated to the development of CHF.

At a previous visit, the parents of an infant with cystic fibrosis received instruction in the administration of pancrelipase (Pancrease). At a follow-up visit, which finding in the infant suggests that the parents require more teaching about administering the pancreatic enzymes? 1. Fatty stools 2. Liquid stools 3. Bloody stools 4. Normal stools

1. Fatty stools RATIONALE: Pancreatic enzymes normally aid in food digestion in the intestine. In a child with cystic fibrosis, however, these natural enzymes cannot reach the intestine because mucus blocks the pancreatic duct. Without these enzymes, undigested fats and proteins produce fatty stools. If the parents were administering the pancreatic enzymes correctly, the child would have stools of normal consistency. Noncompliance doesn't cause liquid or bloody stools.

A nurse is reviewing a health care provider's prescription for a child who was just admitted to the hospital with a diagnosis of Kawasaki disease. Which prescription should the nurse anticipate being part of the treatment plan? 1. Immune globulin 2. Heparin infusion 3. Morphine sulfate 4. Digoxin (Lanoxin)

1. Immune globulin Rationale: Intravenous immune globulin (IVIG) is administered to the child with Kawasaki disease to decrease the incidence of coronary artery lesions and aneurysms and to decrease fever and inflammation. Options 2, 3, and 4 are not components of the treatment plan for this disease.

A mother of an infant diagnosed with Hirschsprung's disease asks the nurse about the disorder. The nurse plans to base the response on which of the following? 1. It is a congenital aganglionosis or megacolon. 2. It is a complete small intestinal obstruction. 3. It is a condition that causes the pyloric valve to remain open. 4. It is a severe inflammation of the gastrointestinal tract.

1. It is a congenital aganglionosis or megacolon. Rationale: Hirschsprung's disease, also known as "congenital aganglionosis" or "megacolon," is the result of an absence of ganglion cells in the rectum and to varying degrees upward in the colon. Options 2, 3, and 4 are incorrect.

A nursing student is preparing a clinical conference, and the topic of the discussion is caring for the child with cystic fibrosis (CF). The student prepares a handout for the group and lists which of the following on the handout? Select all that apply. 1. It is a disease that causes mucus formation to be abnormally thick. 2. It is a chronic multisystem disorder affecting the exocrine glands. 3. It is transmitted as an autosomal recessive trait. 4. It is a disease that causes dilation of the passageways of all organs. 5. It is a disease that affects males only. 6. It is a disease that affects the lungs only.

1. It is a disease that causes mucus formation to be abnormally thick. 2. It is a chronic multisystem disorder affecting the exocrine glands. 3. It is transmitted as an autosomal recessive trait. Rationale: CF is a chronic multisystem disorder affecting the exocrine glands. The mucus produced by these glands (particularly those of the bronchioles, small intestine, and pancreatic and bile ducts) is abnormally thick, causing obstruction of the small passageways of these organs. It is transmitted as an autosomal recessive trait and can affect both males and females.

A nursing student is preparing a clinical conference, and the topic of the discussion is caring for the child with cystic fibrosis (CF). The student prepares a handout for the group and lists which of the following on the handout? Select all that apply. 1. It is a disease that causes mucus formation to be abnormally thick. 2. It is a chronic multisystem disorder affecting the exocrine glands. 3. It is transmitted as an autosomal recessive trait. 4. It is a disease that causes dilation of the passageways of all organs. 5. It is a disease that affects males only. 6. It is a disease that affects the lungs only.

1. It is a disease that causes mucus formation to be abnormally thick. 2. It is a chronic multisystem disorder affecting the exocrine glands. 3. It is transmitted as an autosomal recessive trait. Rationale: CF is a chronic multisystem disorder affecting the exocrine glands. The mucus produced by these glands (particularly those of the bronchioles, small intestine, and pancreatic and bile ducts) is abnormally thick, causing obstruction of the small passageways of these organs. It is transmitted as an autosomal recessive trait and can affect both males and females.

A 10-month-old infant with tetralogy of Fallot (TOF) experiences a cyanotic episode. To improve oxygenation during such an episode, the nurse should place the infant in which position? 1. Knee-to-chest 2. Fowler's 3. Trendelenburg's 4. Prone

1. Knee-to-chest RATIONALE: TOF involves four defects: pulmonary stenosis, right ventricular hypertrophy, ventricular-septal defect (VSD), and dextroposition of the aorta with overriding of the VSD. Pulmonary stenosis decreases pulmonary blood flow and right-to-left shunting via the VSD, causing desaturated blood to circulate. The nurse should place the child in the knee-to-chest position because this position reduces venous return from the legs and increases systemic vascular resistance, maximizing pulmonary blood flow and improving oxygenation status. Fowler's, Trendelenburg's, and the prone positions don't improve oxygenation.

A 15-month-old toddler has just received his routine immunizations, including diphtheria, tetanus, and acellular pertussis; inactivated polio vaccine; measles, mumps, and rubella; varicella; and pneumococcal conjugate vaccine. What information should the nurse give to the parents before they leave the office? Select all that apply. 1. Minor symptoms can be treated with acetaminophen (Tylenol). 2. Minor symptoms can be treated with aspirin (A.S.A.). 3. Call the office if the toddler develops a temperature above 103° F (39.4° C), seizures, or difficulty breathing. 4. Soreness at the immunization site and mild fever are common. 5. The toddler should restrict his activity for the remainder of the day.

1. Minor symptoms can be treated with acetaminophen (Tylenol). 3. Call the office if the toddler develops a temperature above 103° F (39.4° C), seizures, or difficulty breathing. 4. Soreness at the immunization site and mild fever are common. RATIONALE: The nurse should tell the parents that minor symptoms, such as soreness at the immunization site and mild fever, can be treated with acetaminophen or ibuprofen. Aspirin should be avoided in children because of its association with Reye's syndrome. The parents should notify the clinic if serious complications (such as a temperature above 103° F, seizures, or difficulty breathing) occur. Minor discomforts, such as soreness and mild fever, are common after immunizations. Although the child may prefer to rest after immunizations, it isn't necessary to restrict his activity.

A nurse formulates a nursing diagnosis of Risk for infection for a child with Down syndrome. Which condition typically seen in children with this syndrome supports this nursing diagnosis? 1. Muscular hypotonicity 2. Muscle spasticity 3. Increased mucus viscosity 4. Hypothyroidism

1. Muscular hypotonicity RATIONALE: Several conditions make the child with Down syndrome highly vulnerable to respiratory infections. For example, the hypotonicity of chest muscles in children with Down syndrome leads to diminished respiratory expansion and pooling of secretions, and an underdeveloped nasal bone impairs mucus drainage. Down syndrome isn't associated with muscle spasticity or increased mucus viscosity. Although hypothyroidism is common in children with Down syndrome, it doesn't increase the risk of infection.

A nurse is monitoring a 7-year-old child who sustained a head injury in a motor vehicle accident for signs of increased intracranial pressure (ICP). The nurse assesses the child frequently for which early sign of increased ICP? 1. Nausea 2. Papilledema 3. Decerebrate posturing 4. Alterations in pupil size

1. Nausea Rationale: Nausea is an early sign of increased ICP. Late signs of increased ICP include a significant decrease in level of consciousness, Cushing's triad (increased systolic blood pressure and widened pulse pressure, bradycardia, and irregular respirations), and fixed and dilated pupils. Other late signs include decreased motor response to command, decreased sensory response to painful stimuli, posturing, Cheyne-Stokes respirations, and papilledema.

What signs and symptoms would alert the nurse to the possibility of intussusception? Select all that apply. 1. Onset is sudden 2. Kicking and drawing of legs 3. Failure to thrive 4. Bile stained vomit 5. Currant jelly stools

1. Onset is sudden 2. Kicking and drawing of legs 4. Bile stained vomit 5. Currant jelly stools All of these are signs of intussusception except failure to thrive. This diagnosis is an acute condition and it will be diagnosed before a child has the opportunity to develop a diagnosis of failure to thrive.

A nurse is initiating seizure precautions for a child being admitted to the nursing unit. Which of the following items should the nurse place at the bedside?

1. Oxygen and a tongue depressor 2. A suction apparatus and oxygen 3. An airway and a tracheotomy set 4. An emergency cart and an oxygen mask 2. A suction apparatus and oxygen Rationale: Seizures cause a tightening of all body muscles that is followed by tremors. An obstructed airway and increased oral secretions are the major complications during and after the seizure. Suctioning and oxygen are helpful to prevent choking and cyanosis. Option 1 is incorrect-a tongue depressor is not needed and nothing is placed into the client's mouth during a seizure because of the risk for injury. Option 3 is incorrect, because inserting a tracheostomy is not done. Option 4 is incorrect, because an emergency cart would not be left at the bedside- however, it would be available in the treatment room or on the nursing unit.

A 5-year-old child is admitted to the hospital for heart surgery to repair tetralogy of Fallot. The nurse notes that the child has clubbed fingers, and the nurse knows that this symptom is likely a result of: 1. Peripheral hypoxia 2. Chronic hypertension 3. Delayed physical growth 4. Destruction of bone marrow

1. Peripheral hypoxia Rationale: Clubbing, a thickening and flattening of the tips of the fingers and toes, is thought to occur because of a chronic tissue hypoxemia and polycythemia. Options 2, 3, and 4 are not causes of clubbing.

A child with a diagnosis of pertussis (whooping cough) is being admitted to the pediatric unit. As soon as the child arrives in the unit, the nurse would first: 1. Place the child on a pulse oximeter. 2. Weigh the child. 3. Take the child's temperature. 4. Ask the parents about the child.

1. Place the child on a pulse oximeter. Rationale: To adequately determine whether the child is getting enough oxygen, the child is placed on a pulse oximeter. The pulse oximeter will then provide ongoing information on the child's oxygen level. The child is also immediately placed on a cardiorespiratory monitor to provide early identification of periods of apnea and bradycardia. The nurse would then gather data including taking the child's temperature and weight and asking the parents about the child.

A nurse is preparing for the admission of an infant with a diagnosis of bronchiolitis caused by the respiratory syncytial virus (RSV). Choose the interventions that would be included in the plan of care. Select all that apply. 1. Place the infant in a private room. 2. Place the infant in a room near the nurses' station. 3. Ensure that the infant's head is in a flexed position. 4. Wear a mask at all times when in contact with the infant. 5. Place the child in a tent that delivers warm, humidified air. 6. Position the infant side-lying, with the head lower than the chest.

1. Place the infant in a private room. 2. Place the infant in a room near the nurses' station. Rationale: The infant with RSV should be isolated in a private room or in a room with another child with RSV. The infant should be placed in a room near the nurses' station for close observation. The infant should be positioned with the head and chest at a 30- to 40-degree angle and the neck slightly extended to maintain an open airway and to decrease pressure on the diaphragm. Cool, humidified oxygen is delivered to relieve dyspnea, hypoxemia, and insensible water loss from tachypnea. Contact precautions (wearing gloves and a gown) reduce the nosocomial transmission of RSV.

A licensed practical nurse is providing care for a child with hydrocephalus who has had a ventriculoperitoneal shunt revision. Which data collection finding should be reported to the registered nurse immediately? 1. Temperature 100.9° F 2. Pulse 78 beats per minute 3. Blood pressure 110/70 mm Hg 4. Respirations 22 breaths per minute

1. Temperature 100.9 F Rationale: Fever may be an indication of an infection of the shunt, which is the primary concern in the postoperative period, related to a shunt insertion. All of the other vital signs are normal findings for this child.

An 8-year-old boy is being treated with percussion treatments for cystic fibrosis. How would the nurse determine whether the treatment is effective? 1. The child has a productive cough of thick sputum. 2. The child no longer has a fever. 3. The child's skin is no longer high in sodium. 4. The child's bowel movements are firmer.

1. The child has a productive cough of thick sputum. Rationale: Percussion treatments are intended to produce sputum. Thick sputum is characteristic of cystic fibrosis. Being afebrile is not necessarily reflective of effectiveness of percussion treatments. Although a high sodium content in the skin is a sign associated with cystic fibrosis, percussion treatments will not help this characteristic. The percussion treatments will not help bowel movements.

A nurse is caring for a 17-year-old girl with cystic fibrosis who has been admitted to the hospital to receive antibiotics and respiratory treatment for exacerbation of a lung infection. The girl has a number of questions about her future and the consequences of the disease. Which statement about the course of cystic fibrosis is true? 1. The client is at risk for developing diabetes. 2. Pregnancy and child-bearing aren't affected. 3. Only males carry the gene for the disease. 4. By age 20, the client should be able to decrease the frequency of respiratory treatment.

1. The client is at risk for developing diabetes. RATIONALE: Clients with cystic fibrosis are at risk for developing diabetes mellitus because the pancreatic duct becomes obstructed as pancreatic tissues are destroyed. Clients with cystic fibrosis can expect to have normal sexual relationships, but fertility becomes difficult because thick secretions obstruct the cervix and block sperm entry. Males and females carry the gene for cystic fibrosis. Pulmonary disease commonly progresses as the client ages, requiring additional respiratory treatment — not less.

The nurse is teaching a postpartum client about breast-feeding. Which instruction should the nurse include? 1. The diet should include additional fluids. 2. Prenatal vitamins should be discontinued. 3. Soap should be used to cleanse the breasts. 4. Birth control measures are unnecessary while breast-feeding.

1. The diet should include additional fluids

The nurse should implement which of the following in the care of a child who is having a seizure? Select all that apply. 1. Time the seizure. 2. Restrain the child. 3. Stay with the child. 4. Insert an oral airway. 5. Place the child in a supine position. 6. Loosen clothing around the child's neck.

1. Time the seizure 3. Stay with the child 6. Loosen clothing around the child's neck Rationale: During a seizure, the child is placed on his or her side in a lateral position. Positioning on the side will prevent aspiration because saliva will drain out of the corner of the child's mouth. The child is not restrained because this could cause injury to the child. The nurse would loosen clothing around the child's neck and ensure a patent airway. Nothing is placed into the child's mouth during a seizure because this action may cause injury to the child's mouth, gums, or teeth. The nurse would stay with the child to reduce the risk of injury and allow for observation and timing of the seizure.

A nurse is developing a plan of care for a child who is at risk for seizures. Which interventions apply if the child has a seizure? *Select all that apply. 1. Time the seizure.* 2. Restrain the child. 3. Stay with the child.* 4. Place the child in a prone position. 5. Move furniture away from the child.* 6. Insert a padded tongue blade into the child's mouth.

1. Time the seizure. 3. Stay with the child. 5. Move furniture away from the child. Rationale: During a seizure, the child is placed on his or her side in a lateral position. This type of positioning will prevent aspiration, because saliva will drain out of the corner of the child's mouth. The child is not restrained, because this could cause injury. The nurse would loosen clothing around the child's neck and ensure a patent airway. Nothing is placed into the child's mouth during a seizure, because this action may cause injury to the child's mouth, gums, or teeth. The nurse would stay with the child to reduce the risk of injury and allow for the observation and timing of the seizure.

Before performing an otoscopic examination on a child, where should the nurse palpate for tenderness? 1. Tragus, mastoid process, and helix 2. Helix, umbo, and tragus 3. Tragus, cochlea, and lobule 4. Mastoid process, incus, and malleus

1. Tragus, mastoid process, and helix RATIONALE: Before inserting the otoscope, the nurse should palpate the child's external ear, especially the tragus and mastoid process, and should pull the helix backward to determine the presence of pain or tenderness. The umbo, incus, and malleus (parts of the middle ear) and the cochlea (part of the inner ear) aren't palpable.

An infant with congestive heart failure (CHF) is receiving diuretic therapy, and the nurse is closely monitoring the intake and output (I&O). Which is the best method for the nurse to use to monitor the urine output? 1. Weighing the diapers 2. Inserting a Foley catheter 3. Comparing intake with output 4. Measuring the amount of water added to formula

1. Weighing the diapers Rationale: The best method to monitor urine output in an infant on diuretic therapy is to weigh the diapers. The weight of dry diapers is subtracted from the weight of wet diapers to determine the amount of urine excreted: 1 g is equivalent to 1 mL of urine. Comparing intake with output would not provide an accurate measure of urine output. Measuring the amount of water added to formula is unrelated to the amount of output. Although Foley catheter drainage is most accurate in determining output, it is not the best method and places the infant at risk for infection.

A nurse is preparing to administer digoxin (Lanoxin) to an infant with congestive heart failure (CHF). Before administering the medication, the nurse double-checks the dose, counts the apical heart rate for 1 full minute, and obtains a rate of 88 beats per minute. Based on this finding, which of the following is the appropriate nursing action? 1. Withhold the medication. 2. Administer the medication. 3. Double-check the apical heart rate and administer the medication. 4. Check the blood pressure and respirations and administer the medication.

1. Withhold the medication Rationale: Digoxin is effective within a narrow therapeutic range (0.5 to 2 ng/mL). Safety in dosing is achieved by double-checking the dose and counting the apical heart rate for 1 full minute. If the heart rate is less than 100 beats per minute in an infant, the nurse would withhold the dose and notify the registered nurse and health care provider. Options 2, 3, and 4 are incorrect actions.

A toddler is brought to the emergency department with sudden onset of abdominal pain, vomiting, and stools that look like red currant jelly. To confirm intussusception, the suspected cause of these findings, the nurse expects the physician to order: 1. a barium enema. 2. suprapubic aspiration. 3. nasogastric (NG) tube insertion. 4. indwelling urinary catheter insertion.

1. a barium enema. RATIONALE: A nurse should expect the physician to order a barium enema because this test is commonly used to confirm and correct intussusception. Performing a suprapubic aspiration or inserting an NG tube or an indwelling urinary catheter wouldn't help diagnose or treat this disorder.

A 7-year-old boy is hospitalized with cystic fibrosis. To help him manage secretions and avoid respiratory distress, the nurse should: 1. perform chest physiotherapy every 4 hours. 2. give pancreatic enzymes as ordered. 3. place the child in an oxygen tent and have oxygen administered continuously. 4. serve a high-calorie diet.

1. perform chest physiotherapy every 4 hours. RATIONALE: The nurse should perform chest physiotherapy because it aids in loosening secretions in the entire respiratory tract. Pancreatic enzymes aid in the absorption of necessary nutrients — not in managing secretions. Oxygen therapy doesn't aid in loosening secretions and can cause carbon dioxide retention and respiratory distress in children with cystic fibrosis. A high-calorie diet is appropriate but doesn't facilitate respiratory effort.

A 9-year-old child is admitted to the pediatric unit for treatment of cystic fibrosis. A nurse assessing the child's respiratory status should expect to identify: 1. production of thick, sticky mucus 2. harsh, nonproductive cough 3. stridor 4. unilateral decrease in breath sounds

1. production of thick, sticky mucus RATIONALE: Cystic fibrosis is associated with the production of thick, sticky mucus. Cystic fibrosis isn't associated with harsh, nonproductive coughing or with stridor or unilateral decrease in breath sounds.

An infant who weighs 7.5 kg is to receive ampicillin (Omnipen) 25 mg/kg I.V. every 6 hours. How many milligrams should the nurse administer per dose? Record your answer using one decimal place.

187.5 milligrams RATIONALE: The nurse should calculate the correct dose using the following equation: 25 mg/kg × 7.5 kg = 187.5 mg

Symptoms of an earache in an infant include: 1. external drainage, pain, decrease in temperature 2. tugging at the ear and rolling head from side to side 3. crying and pointing to the affected ear 4. redness of the cheeks and cyanosis of the ear

2 Earaches in infants may be manifested by general irritability, frequent rubbing or pulling at the ear, and rolling of the head from side to side.

A practice that has been helpful in preventing intellectual disability is: 1. administering the Stanford- Binet test 2. a blood test at birth 3. careful preschool developmental screening 4. a urine test at age 6 months

2 Nurses can contribute to preventing disorders by promoting genetic counseling, immunizations, newborn screening, and good prenatal care. Nurses can also reiterate the need for early assessment and treatment.

A 4-day-old newborn is receiving phototherapy at home for a bilirubin level of 14 mg/dL. The nurse should plan to include which instruction in the teaching plan of care during the home visit to the mother of the newborn? 1. Applying lotions to exposed newborn skin 2. Assessing skin integrity and fluid status of the newborn 3. Having minimal contact with the newborn to prevent stimulation 4. Advising the mother to limit the newborn's oral intake during phototherapy

2 Phototherapy is the use of intense fluorescent lights to reduce serum bilirubin levels in the newborn. Assessing skin integrity and fluid status of the newborn infant is an essential component of phototherapy. Lotions are not used to ensure the therapeutic effect of light exposure in subcutaneous tissue. Contact with the newborn infant is important. Adequate oral fluids are essential to prevent dehydration because diarrhea is a common side effect of therapy. In addition, safe care for the newborn infant during phototherapy requires shielding the eyes with a soft eye shield to prevent retinal damage, keeping the newborn's skin exposed except for the wearing of a diaper, and changing the newborn's position frequently.

A client who is positive for human immunodeficiency virus (HIV) delivers a newborn infant. The nurse provides instructions to help the client regarding care of her infant. Which client statement indicates the need for further instruction? 1. "I will be sure to wash my hands before and after bathroom use." 2. "I need to breast-feed, especially for the first 6 weeks postpartum." 3. "Support groups are available to assist me with understanding my diagnosis of HIV." 4. "My newborn infant should be on antiviral medications for the first 6 weeks after delivery."

2 The mode of perinatal transmission of human immunodeficiency virus (HIV) to the fetus or neonate of an HIV-positive woman can occur during the antenatal, intrapartal, or postpartum period. HIV transmission can occur during breast-feeding. HIV-positive clients should be encouraged to bottle-feed their infants per the health care provider's prescription. Frequent hand-washing is encouraged. Support groups and community agencies can be identified to assist the parents with the newborn infant's home care, the impact of the diagnosis of HIV infection, and available financial resources. It is recommended that infants of HIV-positive clients receive antiviral medications for the first 6 weeks of life

An assessment of a child reveals deficits in communication and social interaction. The child tends to engage in repetitive behaviors such as arranging and rearranging toys. Based on this assessment, the healthcare provider suspects which of these disorders? 1) cerebral palsy 2) autism spectrum disorder 3) down's syndrome 4) concusion

2) autism

A nurse is providing dietary instructions to the mother of a child with celiac disease. Which statement by the mother indicates a need for further instructions? 1. "I can give my child rice." 2. "I am so pleased that I won't have to eliminate oatmeal from my child's diet." 3. "My child loves corn. I will be sure to include corn in the diet." 4. "I will be sure to give my child vitamin supplements every day."

2. "I am so pleased that I won't have to eliminate oatmeal from my child's diet." Rationale: Dietary management is the mainstay of treatment for the child with celiac disease. All wheat, rye, barley, and oats should be eliminated from the diet and replaced with corn and rice. Vitamin supplements, especially fat-soluble vitamins and folate, may be needed in the early period of treatment to correct deficiencies.

A nurse is teaching the mother of a 5-month-old infant diagnosed with bronchiolitis. Which statement by the mother indicates that teaching has been effective? 1. "I hope my baby will come home from the hospital." 2. "I know that this disease is serious and can lead to asthma." 3. "My baby needs to be cured this time so it won't happen again." 4. "My baby has been sick. A machine will help him breathe."

2. "I know that this disease is serious and can lead to asthma." RATIONALE: By saying bronchiolitis places the child at risk for developing asthma, the mother demonstrates understanding of her infant's condition. If diagnosed and treated promptly, most infants recover from the illness and return home. Infants typically don't have recurrences of bronchiolitis. Infants diagnosed with bronchiolitis rarely require mechanical ventilation.

A teenage mother brings her 1-year-old child to the pediatrician's office for a well-baby checkup. She says that her infant can't sit alone or roll over. An appropriate response by the nurse would be: 1. "This is very abnormal. Your child must be sick." 2. "Let's see about further developmental testing." 3. "Don't worry, this is normal for her age." 4. "Maybe you just haven't seen her do it."

2. "Let's see about further developmental testing." RATIONALE: Stating that further developmental testing is necessary is appropriate because at age 12 months a child should be sitting up and rolling over. Therefore, this child may have developmental problems. Saying the infant's behavior is abnormal or suggesting that the mother hasn't seen her infant do these milestones isn't therapeutic and can cut off communication with the mother. Telling the mother that the infant's behavior is normal misleads the mother with false reassurance.

A nurse should begin screening for lead poisoning when a child reaches which age? 1. 6 months 2. 12 months 3. 18 months 4. 24 months

2. 12 months RATIONALE: The nurse should start screening a child for lead poisoning at age 12 months and perform repeat screenings at 24 months. High-risk infants, such as premature infants and formula-fed infants not receiving iron supplementation, should be screened for iron deficiency anemia at age 6 months. Regular dental visits should begin at age 24 months.

The parents of a child with cystic fibrosis, an autosomal recessive disorder, are considering having a second child. Each parent is heterozygous for the cystic fibrosis trait. What is the chance that their second child will manifest the disorder? 1. 0% 2. 25% 3. 50% 4. 100%

2. 25% RATIONALE: To manifest, or express, an autosomal recessive disorder, a child must inherit the trait from both parents. A heterozygous person carries one normal gene and one affected gene and doesn't express the disorder. Therefore, a child of two heterozygous parents has a one-in-four (25%) chance of manifesting an autosomal recessive disorder. Also, outcomes of previous pregnancies don't influence the probability of subsequent offspring expressing the genetic disorder.

A 1-year-old child is diagnosed with intussusception. The mother of the child asks the nurse to describe the disorder. The nurse tells the mother that this disorder is: 1. An acute bowel obstruction 2. A condition in which a proximal segment of the bowel prolapses into a distal segment of the bowel 3. A condition in which a distal segment of the bowel prolapses into a proximal segment of the bowel 4. A condition that causes an acute inflammatory process in the bowel

2. A condition in which a proximal segment of the bowel prolapses into a distal segment of the bowel Rationale: Intussusception occurs when a proximal segment of the bowel prolapses into a distal segment of the bowel. It is a common cause of acute bowel obstruction in infants and young children. It is not an inflammatory process.

A nurse is preparing to teach a 13-year-old adolescent with asthma to administer his own breathing treatments. Which principle should the nurse keep in mind when planning the teaching session? 1. Adolescents are unable to follow detailed instructions. 2. Adolescents are worried about appearing different from their peers. 3. Adolescents' fine motor coordination isn't sufficiently developed to administer treatments. 4. Adolescents have a well-developed sense of self-identity.

2. Adolescents are worried about appearing different from their peers. RATIONALE: Adolescents have a strong need to belong, and they seek social approval from their peers. Knowing this information will help the nurse construct an effective teaching plan. Adolescents are capable of following detailed instructions. According to Piaget, adolescents are at the formal operations stage and are capable of deductive, reflective, and hypothetical reasoning. Fine motor coordination is well developed by adolescence. According to Erikson's stages of psychosocial development, adolescence is the stage of identity versus role confusion. During this stage, the adolescent strives to establish a sense of identity, identity isn't already well-developed.

A nurse is preparing to teach a 13-year-old adolescent with asthma to administer his own breathing treatments. Which principle should the nurse keep in mind when planning the teaching session? 1. Adolescents are unable to follow detailed instructions. 2. Adolescents are worried about appearing different from their peers. 3. Adolescents' fine motor coordination isn't sufficiently developed to administer treatments. 4. Adolescents have a well-developed sense of self-identity.

2. Adolescents are worried about appearing different from their peers. RATIONALE: Adolescents have a strong need to belong, and they seek social approval from their peers. Knowing this information will help the nurse construct an effective teaching plan. Adolescents are capable of following detailed instructions. According to Piaget, adolescents are at the formal operations stage and are capable of deductive, reflective, and hypothetical reasoning. Fine motor coordination is well developed by adolescence. According to Erikson's stages of psychosocial development, adolescence is the stage of identity versus role confusion. During this stage, the adolescent strives to establish a sense of identity; identity isn't already well-developed.

The pediatric nurse is admitting a child diagnosed with tuberculosis. What personal protective equipment (PPE) will the nurse prepare? 1. Contact precautions 2. Airborne infection isolation 3. Droplet isolation 4. Enteric precautions

2. Airborne infection isolation Airborne infection isolation precautions are used for a patient with conditions such as tuberculosis, varicella, and rubeola.

A nurse is prepared to perform a fundal assessment on a postpartum client. The initial nursing action in performing this assessment is which of the following? 1. Ask the client to turn on her side. 2. Ask the client to urinate and empty her bladder. 3. Massage the fundus gently before determining the level of the fundus. 4. Ask the client to lie flat on her back with the knees and legs flat and straight.

2. Ask the client to urinate and empty her bladder.

A nurse is caring for an adolescent girl who was admitted to the hospital's medical unit after attempting suicide by ingesting acetaminophen (Tylenol). The nurse should incorporate which interventions into the care plan for this girl? Select all that apply. 1. Limit care until the girl initiates a conversation. 2. Ask the girl's parents if they keep firearms in their home. 3. Ask the girl if she's currently having suicidal thoughts. 4. Assist the girl with bathing and grooming as needed. 5. Inspect the girl's mouth after giving oral medications. 6. Assure the girl that anything she says will be held in strict confidence.

2. Ask the girl's parents if they keep firearms in their home. 3. Ask the girl if she's currently having suicidal thoughts. 4. Assist the girl with bathing and grooming as needed. 5. Inspect the girl's mouth after giving oral medications. RATIONALE: Safety is the primary consideration when caring for suicidal clients. Because firearms are the most common method used in suicides, the girl's parents should be encouraged to remove firearms from the home, if applicable. Safety also includes assessing for current suicidal ideation. In many cases, clients who are suicidal are depressed and don't have the energy to care for themselves, so the client may need assistance with bathing and grooming. Because depressed and suicidal clients may hide pills in their cheeks, the nurse should inspect the girl's mouth after giving oral medications. Rather than limit care, the nurse should try to establish a trusting relationship through nursing interventions and therapeutic communication. The girl can't be assured of confidentiality when self-destructive behavior is an issue.

A nurse is caring for a child with a diagnosis of intussusception. Which of the following symptoms would the nurse expect to note in this child? 1. Watery diarrhea 2. Blood and mucus in the stools 3. Profuse projectile vomiting 4. Ribbon-like stools

2. Blood and mucus in the stools Rationale: The child with intussusception classically presents with severe abdominal pain that is crampy and intermittent and that causes the child to draw in his or her knees to the chest. Vomiting may be present, but it is not projectile. Bright red blood and mucus are passed through the rectum and commonly described as currant jelly-like stools. Ribbon-like stools are not a manifestation of this disorder.

A mother arrives at the emergency department with her 5-year-old child and states that the child fell off a bunk bed. A head injury is suspected, and a nurse is monitoring the child continuously for signs of increased intracranial pressure (ICP). Which of the following is a late sign of increased ICP in this child? 1. Nausea 2. Bradycardia 3. Bulging fontanel 4. Dilated scalp veins

2. Bradycardia Rationale: Late signs of increased ICP include a significant decrease in the level of consciousness, bradycardia, and fixed and dilated pupils. Nausea is an early sign of increased ICP. A bulging fontanel and dilated scalp veins are early signs of increased ICP and would be noted in an infant rather than in a 5-year-old child.

The postpartum nurse is providing instructions to the mother of a newborn with hyperbilirubinemia who is being breast fed. The nurse should provide which most appropriate instruction to the mother? 1. Feed the newborn less frequently 2. Continue to breast-feed every 2-4 hours 3. Switch to bottle-feeding the infant for 2 weeks 4. Stop breast-feeding and switch to bottle feeding permanently

2. Continue to breast-feed every 2-4 hours

A child is admitted to the emergency department with suspected intussusception. What significant assessment supports this diagnosis? 1. A gradual onset of pain 2. Currant jelly stools 3. Frothy, bulky stools 4. Vague abdominal pain

2. Currant jelly stools In typical cases the onset is sudden. The child feels severe pain in the abdomen. The child vomits. The stomach contents are green or greenish yellow (bilious). Movements of blood and mucus that contain no feces are common about 12 hours after the onset of the obstruction; these are termed currant jelly stools.

The physician suspects tracheoesophageal fistula in a 1-day-old neonate. Which nursing intervention is most appropriate for this child? 1. Avoiding suctioning unless cyanosis occurs 2. Elevating the neonate's head and giving nothing by mouth 3. Elevating the neonate's head for 1 hour after feedings 4. Giving the neonate only glucose water for the first 24 hours

2. Elevating the neonate's head and giving nothing by mouth RATIONALE: Because of the risk of aspiration, a neonate with a known or suspected tracheoesophageal fistula should be kept with the head elevated at all times and should receive nothing by mouth (NPO). The nurse should suction the neonate regularly to maintain a patent airway and prevent pooling of secretions. Elevating the neonate's head after feedings or giving glucose water are inappropriate because the neonate must remain on NPO status.

A child, age 5, is hospitalized for treatment of Kawasaki disease. Which nursing action best identifies potential complications of this disease? 1. Auscultating breath sounds 2. Instituting cardiac monitoring 3. Monitoring blood pressure 4. Assessing the skin daily

2. Instituting cardiac monitoring RATIONALE: Kawasaki disease sometimes causes cardiac complications, including arrhythmias. Therefore, instituting cardiac monitoring is the best action for detecting such complications. Auscultating for breath sounds, monitoring blood pressure, and assessing the skin daily are also important but not as important as cardiac monitoring.

A mother brings her child to the clinic because the child has developed a rash on the trunk and scalp. The child is diagnosed with varicella. The mother inquires about the infectious period associated with varicella, and the nurse tells the mother that the infectious period: 1. Is unknown 2. Is 1 to 2 days before the onset of the rash to 5 days after the onset of lesions and the crusting of lesions 3. Is 10 days before the onset of symptoms to 15 days after the rash appears 4. Ranges from 2 weeks or less up to several months

2. Is 1 to 2 days before the onset of the rash to 5 days after the onset of lesions and the crusting of lesions Rationale: Varicella is known as chickenpox. The infectious period for varicella is 1 to 2 days before the onset of the rash to 5 days after the onset of lesions and the crusting of lesions. In roseola, the infectious period is unknown. Option 3 describes rubella. Option 4 describes diphtheria.

A nurse is caring for a child with a diagnosis of Kawasaki disease. The mother of the child asks the nurse about the disorder. Which statement most accurately describes Kawasaki disease? 1. It is an acquired cell-mediated immunodeficiency disorder. 2. It is also called mucocutaneous lymph node syndrome and is a febrile generalized vasculitis of unknown cause. 3. It is a chronic multi-system autoimmune disease characterized by the inflammation of connective tissue. 4. It is an inflammatory autoimmune disease that affects the connective tissue of the heart, joints, and subcutaneous tissues.

2. It is also called mucocutaneous lymph node syndrome and is a febrile generalized vasculitis of unknown cause. Rationale: Kawasaki disease, also called mucocutaneous lymph node syndrome, is a febrile generalized vasculitis of unknown etiology. Option 1 describes human immunodeficiency virus (HIV) infection. Option 3 describes systemic lupus erythematosus. Option 4 describes rheumatic fever.

A nurse is reviewing a teaching plan with parents of an infant undergoing repair for a cleft lip. Which instructions are the most appropriate for the nurse to give? Select all that apply. 1. Offer a pacifier as needed. 2. Lay the infant on his back or side to sleep. 3. Sit the infant up for each feeding. 4. Loosen the arm restraints every 4 hours. 5. Clean the suture line after each feeding by dabbing it with saline solution. 6. Give the infant extra care and support.

2. Lay the infant on his back or side to sleep. 3. Sit the infant up for each feeding. 5. Clean the suture line after each feeding by dabbing it with saline solution. 6. Give the infant extra care and support. RATIONALE: The nurse should instruct the parents to lay the infant on his back or side to sleep to prevent trauma to the surgery site. She should also instruct them to feed the infant in the upright position with a syringe and attached tubing to prevent stress to the suture line from sucking. In addition, to prevent crusts and scarring, the suture line should be cleaned after each feeding by dabbing it with half-strength hydrogen peroxide or saline solution. The parents should give the infant extra care and support because he can't meet emotional needs by sucking. Extra attention may also prevent crying, which stresses the suture line. Offering a pacifier isn't appropriate. Pacifiers shouldn't be used during the healing process because they stress the suture line. Arm restraints keep the infant's hands away from his mouth. They should be loosened every 2 hours, not every 4 hours.

A nurse is teaching parents about accident prevention for a toddler. Which guideline is most appropriate? 1. Always make the toddler wear a seat belt when riding in a car. 2. Make sure all medications are kept in containers with childproof safety caps. 3. Never leave a toddler unattended on a bed. 4. Teach rules of the road for bicycle safety.

2. Make sure all medications are kept in containers with childproof safety caps. RATIONALE: Making sure all medications are kept in containers with childproof safety caps is the most appropriate guideline because poisoning accidents are common in toddlers owing to the toddler's curiosity and his increasing mobility and ability to climb. When riding in a car, a toddler should be strapped into a car seat, not a seat belt. A seat belt is an appropriate guideline for a school-age child. Never leaving a child alone on a bed is an appropriate guideline for parents of infants. Toddlers already have the ability to climb on and off of beds and other furniture by themselves. Note, however, that toddlers should never be left unattended on high surfaces, such as an examining table in a physician's office. Teaching the rules of the road for bicycle safety is an appropriate safety measure for a school-age child. Toddlers shouldn't be allowed in the road unsupervised.

A nurse is preparing to assess the uterine fundus of a client in the immediate postpartum period. After locating the fundus, the nurse notes that the uterus feels soft and boggy. Which nursing intervention would be most appropriate? 1. Elevate the client's legs 2. Massage the fundus until it is firm 3. Ask the client to turn on her left side 4. Push on the uterus to assist in expanding clots

2. Massage the fundus until it is firm If the uterus is not contracted firmly, the initial intervention is to massage the fundus until it is firm and to express clots that may have accumulated in the uterus.

Which is the earliest sign of esophageal atresia? 1. Mother develops gestational diabetes. 2. Mother develops polyhydramnios. 3. Infant does not pass meconium within 48 hours. 4.Infant appears to drool during feedings.

2. Mother develops polyhydramnios. The mother will develop polyhydramnios in pregnancy. Gestational diabetes does not factor into this diagnosis. If an infant does not pass meconium, it is more pertinent at 24 hours, and it could be a sign of an imperforate anus. The infant does begin to drool during feeding for the diagnosis; however, the polyhydramnios is an earlier sign.

A nurse is developing a plan to teach a mother how to reduce her infant's risk of developing otitis media. Which direction should the nurse include in the teaching plan? 1. Administer antibiotics whenever the infant has a cold. 2. Place the infant in an upright position when giving a bottle. 3. Avoid getting the infant's ears wet while bathing or swimming. 4. Clean the infant's external ear canal daily.

2. Place the infant in an upright position when giving a bottle. RATIONALE: Feeding an infant a bottle in an upright position reduces the pooling of formula or breast milk in the nasopharynx. Formula, in particular, provides a good medium for the growth of bacteria, which can travel easily through the short, horizontal eustachian tubes. Administering antibiotics whenever the infant has a cold, avoiding getting the ears wet, and cleaning the external ear canal daily don't reduce the risk of an infant developing otitis media.

A nurse is caring for a 2½-year-old child with tetralogy of Fallot (TOF). Which abnormalities are associated with TOF? 1. Aortic stenosis, atrial septal defect, overriding aorta, and left ventricular hypertrophy 2. Pulmonic stenosis, intraventricular septal defect, overriding aorta, and right ventricular hypertrophy 3. Pulmonic stenosis, patent ductus arteriosus, overriding aorta, and right ventricular hypertrophy 4. Transposition of the great vessels, intraventricular septal defect, right ventricular hypertrophy, and patent ductus arteriosus

2. Pulmonic stenosis, intraventricular septal defect, overriding aorta, and right ventricular hypertrophy RATIONALE: TOF consists of four congenital anomalies: pulmonic stenosis, intraventricular septal defect, overriding aorta, and right ventricular hypertrophy. The other combinations of defects aren't characteristic of TOF.

A nurse is caring for a 2½-year-old child with tetralogy of Fallot (TOF). Which abnormalities are associated with TOF? 1. Aortic stenosis, atrial septal defect, overriding aorta, and left ventricular hypertrophy 2. Pulmonic stenosis, intraventricular septal defect, overriding aorta, and right ventricular hypertrophy 3. Pulmonic stenosis, patent ductus arteriosus, overriding aorta, and right ventricular hypertrophy 4. Transposition of the great vessels, intraventricular septal defect, right ventricular hypertrophy, and patent ductus arteriosus

2. Pulmonic stenosis, intraventricular septal defect, overriding aorta, and right ventricular hypertrophy RATIONALE: TOF consists of four congenital anomalies: pulmonic stenosis, intraventricular septal defect, overriding aorta, and right ventricular hypertrophy. The other combinations of defects aren't characteristic of TOF.

A 1-year-old child is admitted to the hospital for control of tonic-clonic seizures. The nurse would do which of the following in order to protect the child from injury? Select all that apply. 1. Keep a padded tongue blade at the bedside for use during a seizure. 2. Remove toys that have bright, blinking lights on them. 3. Keep side rails and other hard objects padded. 4. Turn the client to the side during a seizure. 5. Restrict the client's fluid intake.

2. Remove toys that have bright, blinking lights on them. 3. Keep side rails and other hard objects padded. 4. Turn the client to the side during a seizure. Rationale: Attempting to place something in a child's mouth during a seizure is not helpful even if it is padded. The mouth is usually clenched, and one would have to use force to open the mouth. One must attempt to keep the airway clear and can do that by positioning (option 4). Option 2 may be helpful in preventing a seizure, and option 3 safeguards the client's physical safety. Option 5 is not necessary.

A nurse is caring for a young child with tetralogy of Fallot (TOF). The child is upset and crying. The nurse observes that he's dyspneic and cyanotic. Which position would help relieve the child's dyspnea and cyanosis? 1. Sitting in bed with the head of the bed at a 45-degree angle 2. Squatting 3. Lying flat in bed 4. Lying on his right side

2. Squatting RATIONALE: Placing the child in a squatting position sequesters a large amount of blood to the legs, reducing venous return. Sitting with the head of the bed at a 45-degree angle, lying flat, and lying on the right side don't reduce venous return; therefore, they won't relieve the child's dyspnea and cyanosis. A child with TOF may also assume a knee-chest position to reduce venous return to the heart.

Following a cleft lip repair, the nurse provides instructions to the parents regarding cleaning of the lip repair site. Which of the following solutions would the nurse use in demonstrating this procedure to the parents? 1. Tap water 2. Sterile water 3. Full-strength hydrogen peroxide 4. Half-strength hydrogen peroxide

2. Sterile water Rationale: The lip repair site is cleansed with sterile water using a cotton swab after feeding and as prescribed. The parents should be instructed to use a rolling motion from the suture line out. The parents should also demonstrate performance of the correct procedure to the nurse.

A nurse is collecting data on a 12-month-old child with iron deficiency anemia. Which of the following findings would the nurse expect to note in this child? 1. Bradycardia 2. Tachycardia 3. Hyperactivity 4. A reddened appearance to the cheeks

2. Tachycardia Rationale: Clinical manifestations of iron deficiency anemia will vary with the degree of anemia but usually include extreme pallor with porcelain-like skin, tachycardia, lethargy, and irritability.

Parents of a 2-year-old child with chronic otitis media are concerned that the disorder has affected their child's hearing. Which behavior suggests that the child has a hearing impairment? 1. Stuttering 2. Using gestures to express desires 3. Babbling continuously 4. Playing alongside rather than interacting with peers

2. Using gestures to express desires RATIONALE: Using gestures instead of verbal communication to express desires — especially in a child older than age 15 months — may indicate a hearing or communication impairment. Stuttering is normal in children ages 2 to 4, especially boys. Continuous babbling is a normal phase of speech development in young children. In fact, its absence, not presence, would be cause for concern. Parallel play — playing alongside peers without interacting — is typical of toddlers. However, in an older child, difficulty interacting with peers or avoiding social situations may indicate a hearing deficit.

A child, age 4, is admitted with a tentative diagnosis of congenital heart disease. When assessment reveals a bounding radial pulse coupled with a weak femoral pulse, the nurse suspects that the child has: 1. patent ductus arteriosus. 2. coarctation of the aorta. 3. a ventricular septal defect. 4. truncus arteriosus.

2. coarctation of the aorta. RATIONALE: The nurse should suspect coarctation of the aorta because it causes signs of peripheral hypoperfusion, such as a weak femoral pulse and a bounding radial pulse. These signs are rare in patent ductus arteriosus, ventricular septal defect, and truncus arteriosus.

An emergency department nurse suspects neglect in a 3-year-old boy admitted for failure to thrive. Signs of neglect in the child would include: 1. slapping, kicking, and punching others. 2. poor hygiene and weight loss. 3. loud crying and screaming. 4. pulling hair and hitting.

2. poor hygiene and weight loss. RATIONALE: Signs of neglect include poor hygiene and weight loss because neglect can involve failure to provide food, bed, shelter, health care, or hygiene. Slapping, kicking, pulling hair, hitting, and punching are examples of forms of physical abuse, not neglect. Loud crying and screaming are normal findings in a 3-year-old boy.

A school nurse is evaluating a 7-year-old child who is having an asthma attack. The child is cyanotic and unable to speak, with decreased breath sounds and shallow respirations. Based on these physical findings, the nurse should first: 1. monitor the child with a pulse oximeter in her office. 2. prepare to ventilate the child. 3. return the child to class. 4. contact the child's parent or guardian.

2. prepare to ventilate the child. RATIONALE: The nurse should recognize these physical findings as signs and symptoms of impending respiratory collapse. Therefore, the nurse's top priority is to assess airway, breathing, and circulation, and prepare to ventilate the child if necessary. The nurse should then notify the emergency medical systems to transport the child to a local hospital. Because the child's condition requires immediate intervention, simply monitoring pulse oximetry would delay treatment. This child shouldn't be returned to class. When the child's condition allows, the nurse can notify the parents or guardian.

The nurse is assessing a pregnant client in the second trimester of pregnancy who was admitted to the maternity unit with a suspected diagnosis of abruptio placentae. Which assessment finding should the nurse expect to note if this condition is present? 1. Soft abdomen 2. Uterine tenderness 3. absence of abdominal pain 4. painless, bright red vaginal bleeding

2. uterine tenderness

The nurse in the newborn nursery is preparing to complete an initial assessment on a newborn infant who was just admitted to the nursery. The nurse should place a warm blanket on the examining table to prevent heat loss in the infant caused by which method? 1. Radiation 2. Convection 3. Conduction 4. Evaporation

3 Heat loss occurs by four different mechanisms. In conduction, heat loss occurs when the infant is on a cold surface, such as a table. Radiation occurs when heat from the body surface radiates to the surrounding environment. In convection, air moving across the infant's skin transfers heat to the air. Evaporation of moisture from a wet body surface dissipates heat along with the moisture.

A nurse is providing discharge teaching to the parents of a newborn regarding circumcision care. Which of the following statements made by a parent indicates an understanding of the teaching? 1. "His circumcision will heal within 24 hours." 2. "I should remove the yellow mucus that will form." 3."I will clean his penis with each diaper change." 4."I will give him a tub bath within a couple of days."

3. "I will clean his penis with each diaper change"

After a nurse explains dietary restrictions to the parents of a child with celiac disease, which statement by the parents indicates effective teaching? 1. "We'll follow these instructions until our child's symptoms disappear." 2. "Our child must maintain these dietary restrictions until adulthood." 3. "Our child must maintain these dietary restrictions for life." 4. "We'll follow these instructions until our child has completely grown and developed."

3. "Our child must maintain these dietary restrictions for life." RATIONALE: Teaching is effective if the parents say their child must maintain the dietary restrictions for life because the child needs to avoid recurrence of the disease's clinical manifestations. Signs and symptoms will reappear if the client eats prohibited foods later in life.

A physician orders meperidine (Demerol), 30 mg I.M., as preoperative medication for a school-age child who weighs 66 lb (30 kg). The meperidine is supplied as 50 mg/ml. How much meperidine should the nurse administer? 1. 0.3 ml 2. 0.5 ml 3. 0.6 ml 4. 0.8 ml

3. 0.6 ml RATIONALE: By using the fraction method and cross-multiplying to solve for X, the nurse can determine that 0.6 ml should be administered: X ml/30 mg = 1 ml/50 mg X ml × 50 mg = 30 mg × 1 ml X = 0.6 ml.

The nurse is reviewing the record of a client in the labor room and notes that the health care provider had documented the fetal presenting part is at the -1 station. This documented finding indicates that the fetal presenting part is located at which area? 1. 1 inch below the coccyx 2. 1 inch below the iliac crest 3. 1 cm above the ischial spine 4. 1 fingerbreadth below the symphysis pubis

3. 1 cm above the ischial spine

Which item in the care plan for a toddler with a seizure disorder should a nurse revise? 1. Padded side rails 2. Oxygen mask and bag system at bedside 3. Arm restraints while asleep 4. Cardiorespiratory monitoring

3. Arm restraints while asleep RATIONALE: The nurse should revise a care plan that includes restraints. Restraints should never be used on a child with a seizure disorder because they could harm him if a seizure occurs. Padded side rails will prevent the child from injuring himself during a seizure. The bag and mask system should be present in case the child needs oxygen during a seizure. Cardiopulmonary monitoring should be readily available for checking vital signs during a seizure.

A newborn is transferred to the neonatal intensive care unit with an admitting diagnosis of esophageal atresia accompanied by a distal tracheoesophageal fistula (TEF). When assisting to care for the newborn, the priority concern would be: 1. Pain 2. Infection 3. Aspiration 4. The parents' concerns

3. Aspiration Rationale: Because TEF manifests itself with regurgitation and coughing, the concern that has the highest priority is aspiration. Although the other problems are an important part of care, the one with the highest concern relates to airway.

Which of the following is the most appropriate location for assessing the pulse of an infant who is less than 1 year old? 1. Radial 2. Carotid 3. Brachial 4. Popliteal

3. Brachial Rationale: To assess a pulse in an infant (i.e., a child <1 year old), the pulse is checked at the brachial artery. The infant's relatively short, fat neck makes palpation of the carotid artery difficult. The popliteal and radial pulses are also difficult to palpate in an infant.

A nurse reviews the record of an infant who is seen in the clinic. The nurse notes that a diagnosis of esophageal atresia with tracheoesophageal fistula (TEF) is suspected. The nurse expects to note which most likely clinical manifestation of this condition in the medical record? 1. Incessant crying 2. Coughing at nighttime 3. Choking with feedings 4. Severe projectile vomiting

3. Choking with feedings Rationale: Any child who exhibits the "3 Cs"—coughing and choking during feedings and unexplained cyanosis—should be suspected of having TEF. Options 1, 2, and 4 are not specifically associated with TEF.

A nurse is assisting in developing a plan of care for a child admitted with a diagnosis of Kawasaki disease. In developing the initial plan of care, the nurse suggests that the child should be monitored for signs of: 1. Failure to thrive 2. Bleeding 3. Congestive heart failure (CHF) 4. Decreased tolerance to stimulation

3. Congestive heart failure (CHF) Rationale: Nursing care for Kawasaki disease initially centers around observing for signs of CHF. The nurse monitors for increased respiratory rate, increased heart rate, dyspnea, lung congestion, and abdominal distention. Options 1, 2, and 4 are not findings directly associated with this disorder.

A nurse is assisting in developing a plan of care for a child admitted with a diagnosis of Kawasaki disease. In developing the initial plan of care, the nurse suggests that the child should be monitored for signs of: 1. Failure to thrive 2. Bleeding 3. Congestive heart failure (CHF) 4. Decreased tolerance to stimulation

3. Congestive heart failure (CHF) Rationale: Nursing care for Kawasaki disease initially centers around observing for signs of CHF. The nurse monitors for increased respiratory rate, increased heart rate, dyspnea, lung congestion, and abdominal distention. Options 1, 2, and 4 are not findings directly associated with this disorder.

When a nurse assesses a 2-year-old child with suspected dehydration, which condition should be reported to the physician immediately? 1. Irritability for the past 12 hours 2. Capillary refill less than 2 seconds 3. Decreased blood pressure 4. Tachycardia, dry skin, and dry mucous membranes

3. Decreased blood pressure RATIONALE: The nurse should immediately report decreased blood pressure because it's a late sign of severe dehydration. This delayed decrease occurs because compensatory mechanisms in children are able to sustain blood pressure in the low-normal range for some time. Irritability, capillary refill less than 2 seconds, tachycardia, dry skin, and dry mucous membranes are all early signs of dehydration.

A child is admitted to the pediatric unit with a diagnosis of coarctation of the aorta (COA). The health care provider prescribes that the child's blood pressure be taken every 4 hours in the legs and arms. The nurse would expect the blood pressure in the child's legs and arms to be: 1. Increased in the arms and the legs 2. Decreased in the arms and the legs 3. Decreased in the legs and increased in the arms 4. Increased in the legs and decreased in the arms

3. Decreased in the legs and increased in the arms Rationale: Coarctation indicates a narrowing in the aorta. This would indicate an increased pressure proximal to the defect and a decreased pressure distal to the defect. This would result in a lower blood pressure in the legs and a higher blood pressure in the arms, which is indicated in option 3.

A 10-year-old child with asthma is treated for acute exacerbation. Which finding would indicate that the condition is worsening? 1. Warm, dry skin 2. Increased wheezing 3. Decreased wheezing 4. A pulse rate of 90 beats per minute

3. Decreased wheezing Rationale: Decreased wheezing in a child who is not improving clinically may be interpreted incorrectly as a positive sign, when in fact it may signal an inability to move air. A "silent chest" is an ominous sign during an asthma episode. With treatment, increased wheezing may actually signal that the child's condition is improving. Warm, dry skin indicates an improvement in the condition because the child is normally diaphoretic during exacerbation. The normal pulse rate in a 10-year-old is 70 to 110 beats per minute.

A nurse in a delivery room is assisting with the delivery of a newborn. After delivery, the nurse prepares to prevent heat loss in the newborn resulting from evaporation by: 1. Warming the crib pad. 2. Closing the doors to the room. 3. Drying the infant with a warm blanket. 4. Turning on the overhead radiant warmer.

3. Drying the infant with a warm blanket

A nurse reviews the record of a child who was just seen by a health care provider (HCP). The HCP has documented a diagnosis of suspected aortic stenosis. Which clinical manifestation that is specifically found in children with this disorder should the nurse anticipate? 1. Pallor 2. Hyperactivity 3. Exercise intolerance 4. Gastrointestinal disturbances

3. Exercise intolerance Rationale: The child with aortic stenosis shows signs of exercise intolerance, chest pain, and dizziness when standing for long periods. Pallor may be noted, but it is not specific to this type of disorder alone. Options 2 and 4 are not related to this disorder.

A toddler with hemophilia is hospitalized with multiple injuries after falling off a sliding board. X-rays reveal no bone fractures. When caring for the child, what is the nurse's highest priority? 1. Administering platelets as ordered 2. Taking measures to prevent infection 3. Frequently assessing the child's level of consciousness (LOC) 4. Discussing a safe play environment with the parents

3. Frequently assessing the child's level of consciousness (LOC) RATIONALE: In hemophilia, one of the factors required for blood clotting is absent, significantly increasing the risk of hemorrhage after injury. Therefore, the nurse must assess the child frequently for signs and symptoms of intracranial bleeding, such as an altered LOC, slurred speech, vomiting, and headache. To manage hemophilia, the absent blood clotting factor is replaced via I.V. infusion of factor, cryoprecipitate, or fresh frozen plasma; this may be done prophylactically or after a traumatic injury. Platelet transfusions aren't necessary. Clients with hemophilia aren't at increased risk for infection. Discussing a safe play environment with the parents is important but isn't the highest priority.

An adolescent is diagnosed with lead poisoning. After emphasizing the importance of consuming dietary iron, the nurse asks him to select iron-rich breakfast items from a sample menu. Which selection demonstrates knowledge of dietary iron sources? 1. Grapefruit and white toast 2. Pancakes and a banana 3. Ham and eggs 4. Bagel and cream cheese

3. Ham and eggs RATIONALE: Good sources of dietary iron include red meat, egg yolks, whole wheat breads, seafood, nuts, legumes, iron-fortified cereals, and green, leafy vegetables. Fresh fruits and milk products contain only small amounts of iron. White bread isn't a good iron source.

According to Erikson's psychosocial theory of development, an 8-year-old child would be in which stage? 1. Trust versus mistrust 2. Initiative versus guilt 3. Industry versus inferiority 4. Identity versus role confusion

3. Industry versus inferiority RATIONALE: In middle childhood, the 6- to 12-year-old child is mastering the task of industry versus inferiority. The trust versus mistrust task is in infancy (birth to 1 year). In early childhood, the 1- to 3-year-old child is in the stage of initiative versus guilt. Identity versus role confusion occurs

A nurse is assessing a client who is 6 hours post-partum after delivering a full-term healthy infant. The client complains to the nurse of feelings of faintness and dizziness. Which nursing action would be most appropriate? 1. Elevate the client's legs. 2. Determine the hemoglobin and hematocrit levels. 3. Instruct the client to request help when getting out of bed. 4. Inform the nursery room nurse to avoid bringing the newborn infant to the client until the feelings of faintness and dizziness have subsided.

3. Instruct the client to request help when getting out of bed.

Which of the following represents a primary characteristic of autism? 1. Normal social play 2. Consistent imitation of others' actions 3. Lack of social interaction and awareness 4. Normal verbal and nonverbal communication

3. Lack of social interaction and awareness Rationale: Autism is a severe form of an autism spectrum disorder. A primary characteristic is a lack of social interaction and awareness. Social behaviors in autism include a lack of or an abnormal imitation of others' actions and a lack of or abnormal social play. Additional characteristics include a lack of or impaired verbal communication and markedly abnormal nonverbal communication.

A nurse is caring for a school-age child with cerebral palsy. The child has difficulty eating using regular utensils and requires a lot of assistance. Which referral is most appropriate? 1. Registered dietitian 2. Physical therapist 3. Occupational therapist 4. Nursing assistant

3. Occupational therapist RATIONALE: An occupational therapist helps physically disabled clients adapt to physical limitations and is most qualified to help a child with cerebral palsy eat and perform other activities of daily living. A registered dietitian manages and plans for the nutritional needs of children with cerebral palsy but isn't trained in modifying or fitting utensils with assistive devices. A physical therapist is trained to help a child with cerebral palsy gain function and prevent further disability but not to assist the child in performing activities of daily living. A nursing assistant can help a child eat; however, the nursing assistant isn't trained in modifying utensils.

An infant returns to the nursing unit after the surgical repair of a cleft lip located on the right side of the lip. The best position in which to place this infant at this time is: 1. A flat position 2. A prone position 3. On his or her left side 4. On his or her right side

3. On his or her left side Rationale: After the repair of a cleft lip, the infant should be positioned on the side opposite to the repair to prevent contact of the suture lines with the bed linens. In this case, it is best to place the infant on his or her left side. Additionally, the flat or prone position can result in aspiration if the infant vomits.

A nurse is caring for a hospitalized infant with bronchiolitis. Diagnostic tests have confirmed respiratory syncytial virus (RSV). On the basis of this finding, which of the following would be the appropriate nursing action? 1. Initiate strict enteric precautions. 2. Wear a mask when caring for the child. 3. Plan to move the infant to a room with another child with RSV. 4. Leave the infant in the present room, because RSV is not contagious.

3. Plan to move the infant to a room with another child with RSV. Rationale: RSV is a highly communicable disorder, but it is not transmitted via the airborne route. It is usually transferred by the hands, and meticulous handwashing is necessary to decrease the spread of organisms. The infant with RSV is isolated in a single room or placed in a room with another child with RSV. Enteric precautions are not necessary; however, the nurse should wear a gown when the soiling of clothing may occur.

A nurse prepares a teaching plan regarding the administration of eardrops for the parents of a 2-year-old child. Which of the following would be included in the plan? 1. Wear gloves when administering the eardrops. 2. Pull the ear up and back before instilling the eardrops. 3. Pull the earlobe down and back before instilling the ear drops. 4. Hold the child in a sitting position when administering the ear drops.

3. Pull the earlobe down and back before instilling the ear drops. Rationale: When administering eardrops to a child who is less than 3 years old, the ear should be pulled down and back. For children who are more than 3 years old, the ear is pulled up and back. Gloves do not need to be worn by the parents, but handwashing needs to be performed before and after the procedure. The child should be in a side-lying position with the affected ear facing upward to facilitate the flow of medication down the ear canal by gravity.

A toddler is having a tonic-clonic seizure. What should the nurse do first? 1. Restrain the child. 2. Place a tongue blade in the child's mouth. 3. Remove objects from the child's surroundings. 4. Check the child's breathing.

3. Remove objects from the child's surroundings. RATIONALE: During a seizure, the nurse's first priority is to protect the child from injury caused by uncontrolled movements. Therefore, the nurse must first remove objects from the child's surroundings and pad objects that can't be removed. Restraining the child or placing an object in the child's mouth during a seizure isn't appropriate because it may cause injury. When the seizure stops, the nurse should then check for breathing and, if indicated, initiate rescue breathing.

A nurse caring for an infant with congenital heart disease is monitoring the infant closely for signs of congestive heart failure (CHF). The nurse looks for which early sign of CHF? 1. Pallor 2. Cough 3. Tachycardia 4. Slow and shallow breathing

3. Tachycardia Rationale: The early signs of CHF include tachycardia, tachypnea, profuse scalp sweating, fatigue, irritability, sudden weight gain, and respiratory distress. A cough may occur with CHF as a result of mucosal swelling and irritation, but it is not an early sign. Pallor may be noted in the infant with CHF, but it is also not an early sign.

A nurse is providing home care instructions to the mother of a child with bacterial conjunctivitis. The nurse should tell the mother: 1. That the child may attend school if antibiotics have been started 2. To save any unused eye medication in case a sibling gets the eye infection 3. That the child's towels and washcloths should not be used by other members of the household 4. To wipe any crusted material from the eye with a cotton ball soaked in warm water, starting at the outer aspect of the eye and moving toward the inner aspect

3. That the child's towels and washcloths should not be used by other members of the household Rationale: Bacterial conjunctivitis is highly contagious, and infection control measures should be taught. These include good handwashing and not sharing towels and washcloths with others. The child should be kept home from school until 24 hours after antibiotics are started. Bottles of eye medication should never be shared with others. Crusted material may be wiped from the eye with a cotton ball soaked in warm water, starting at the inner aspect of the eye and moving toward the outer aspect.

An emergency department nurse is caring for a child brought to the emergency department following the ingestion of approximately one half bottle of acetylsalicylic acid (aspirin). The nurse anticipates that the likely initial treatment will be: 1. Dialysis 2. The administration of vitamin K 3. The administration of activated charcoal 4. The administration of sodium bicarbonate

3. The administration of activated charcoal Rationale: Initial treatment of salicylate overdose includes administration of activated charcoal to decrease absorption of the aspirin. Intravenous (IV) fluids and inducing emesis may be prescribed to enhance excretion but would not be the initial treatment. Dialysis is used in extreme cases if the child is unresponsive to therapy. Vitamin K is the antidote for warfarin (Coumadin) overdose.

A nurse is caring for a client who is receiving Oxytocin/Pitocin to induce labor. The nurse discontinues the infusion if which of the following is noted on assessment of the client? 1. Fatigue 2. Drowsiness 3. Uterine Hyperstimulation 4. Early decelerations of the fetal heart rate

3. Uterine hyperstimulation

A nurse is caring for a hospitalized infant with a diagnosis of bronchiolitis. The nurse positions the infant: 1. With the head at a 60-degree angle with the neck slightly flexed 2. In a supine, side-lying position 3. With the head and chest at a 30-degree angle, with the neck slightly extended 4. Prone, with the head of the bed elevated 15 degrees

3. With the head and chest at a 30-degree angle, with the neck slightly extended Rationale: The nurse should position the infant with the head and the chest at a 30- to 40-degree angle with the neck slightly extended to maintain an open airway and to decrease pressure on the diaphragm. Options 1, 2, and 4 do not achieve these goals.

A 2-year-old child is admitted to the pediatric unit with fever, seizures, and vomiting. He's awake and alert. As the nurse is putting a gown on the child, the nurse notices petechiae across the child's chest, abdomen, and back. The nurse should: 1. question the mother about the child's allergies. 2. initiate standard precautions. 3. evaluate the child's neurologic status. 4. examine the child's throat and ears.

3. evaluate the child's neurologic status. RATIONALE: Petechiae across the child's chest, abdomen, and back are signs of meningitis. The priority is to evaluate neurologic status. Petechiae aren't allergic reactions, so the nurse shouldn't ask about allergies. Standard precautions should be used when there is risk of contacting body fluids. Contact precautions should be instituted for the client diagnosed with meningitis. Throat and ear examinations wouldn't be helpful in confirming a diagnosis of meningitis.

A client arrives at birthing center in active labor. Her membranes are still intact, and the health care provider prepares to perform an amniotomy. What will the nurse relay to the client as the most likely outcome of the amniotomy? 1. less pressure on her cervix 2. decreased number of contractions 3. increased efficiency of contractions 4. the need for increased maternal blood pressure monitoring

3. increased efficiency of contractions

A 3-month-old infant just had a cleft lip and pallet repair. To prevent trauma to the operative site, the nurse should: 1. give the infant a pacifier to help soothe him. 2. lie the infant in the prone position. 3. place the infant's arms in soft elbow restraints. 4. avoid touching the suture line, even to clean.

3. place the infant's arms in soft elbow restraints. RATIONALE: Soft restraints from the upper arm to the wrist are appropriate because they prevent the infant from touching his lip but allow him to hold a favorite item such as a blanket. Because they could damage the operative site, such objects as pacifiers, suction catheters, and small spoons shouldn't be placed in an infant's mouth after cleft palette repair. An infant in a prone position may rub his face on the sheets and traumatize the operative site. The suture line should be cleaned gently to prevent infection, which could interfere with healing and damage the cosmetic appearance of the repair. Dried blood collecting on the suture line can widen the scar.

The nurse is caring for a client in labor. Which assessment finding indicates to the nurse that the client is beginning the second stage of labor? 1. The contractions are regular. 2. The membranes have ruptured 3. The cervix is completely dilated 4. The client starts to expel clear vaginal fluid

3. the cervix is completely dilated

The nurse is providing instructions to a new mother regarding cord care for a newborn infant. Which statement, if made by the mother, indicates a need for further instructions? 1. "The cord will fall off in 1 to 2 weeks." 2. "Alcohol may be used to clean the cord." 3. "I should cleanse the cord two or three times a day." 4. "I need to fold the diaper above the cord to prevent infection."

4 The diaper should be folded below the cord to keep urine away from the cord, so a statement by the client that the diaper should be folded above the cord would be incorrect, indicating the need for further instruction. The cord should be kept clean and dry to decrease bacterial growth. Cord care is required until the cord dries up and falls off, between 7 and 14 days after birth. The cord should be cleansed two or three times a day with soap and water or other prescribed agents. The doctor may or may not recommend alcohol.

The nurse is preparing to care for a newborn receiving photo-therapy. Which interventions should be included in the plan of care? Select all that apply. 1. Avoid stimulation. 2. Decrease fluid intake. 3. Expose all of the newborn's skin. 4. Monitor skin temperature closely. 5. Reposition the newborn every 2 hours. 6. Cover the newborn's eyes with eye shields or patches.

4, 5, 6 Phototherapy is the use of intense fluorescent lights to reduce serum bilirubin levels in the newborn. Adverse effects from treatment, such as eye damage, dehydration, or sensory deprivation, can occur. Interventions include exposing as much of the newborn's skin as possible; however, the genital area is covered. The newborn's eyes are also covered with eye shields or patches, ensuring that the eyelids are closed when shields or patches are applied. The shields or patches are removed at least once per shift to inspect the eyes for infection or irritation and to allow eye contact. The nurse measures the lamp energy output to ensure efficacy of the treatment (done with a special device known as a photometer), monitors skin temperature closely, and increases fluids to compensate for water loss. The newborn will have loose green stools and green-colored urine. The newborn's skin color is monitored with the fluorescent light turned off every 4 to 8 hours and is monitored for bronze baby syndrome, a grayish brown discoloration of the skin. The newborn is repositioned every 2 hours, and stimulation is provided. After treatment, the newborn is monitored for signs of hyperbilirubinemia because rebound elevations can occur after therapy is discontinued.

A nurse is told that a child with Kawasaki disease will be arriving to the nursing unit for admission. Which question should the nurse ask the family to elicit information specific to the development of Kawasaki disease? 1. "Has the child complained of back pain?" 2. "Has the child complained of headaches?" 3. "Has the child had any nausea or vomiting?" 4. "Did the child unexplained fever within the past 10 days?"

4. "Did the child unexplained fever within the past 10 days?" Kawasaki disease beings with an unexplained fever and the child tends to be otherwise asymptomatic. Fever tends to be 102+ and can last for 5 days.

The mother of an infant newly diagnosed with cystic fibrosis is being taught proper nutritional needs for the infant. The nurse determines that the mother understands nutritional needs when the mother replies: 1. "I know that my infant needs to drink predigested formula until she has her stool pattern developed." 2. "When I begin feeding my infant cereal, I will make sure to warm the cereal and administer the pancreatic enzyme mixed in." 3. "I will make sure that I give my infant fat-free milk as a supplement to her predigested formula, because she is not able to digest fat." 4. "I know I need to monitor my infant's stools and if there are more than four stools a day, I will increase the pancreatic enzyme."

4. "I know I need to monitor my infant's stools and if there are more than four stools a day, I will increase the pancreatic enzyme." Rationale: Cystic fibrosis requires a high-calorie, high-protein diet with pancreatic enzyme replacement therapy. The infant needs to remain on the predigested formula until 1 year of age, when formula can be discontinued and then fat-free milk consumed. The pancreatic enzyme should not be mixed with warmed foods because this inactivates the enzyme. Stools must be monitored, and pancreatic enzymes are administered based on the stool pattern.

A boy, age 3, develops a fever and rash and is diagnosed with rubella. His mother has just given birth to a baby girl. Which statement by the mother best indicates that she understands the implications of rubella? 1. "I told my husband to give my son aspirin for his fever." 2. "I'll ask the physician about giving the baby an immunization shot." 3. "I don't have to worry because I've had the measles." 4. "I'll call my neighbor who's 2 months pregnant and tell her not to have contact with my son."

4. "I'll call my neighbor who's 2 months pregnant and tell her not to have contact with my son." RATIONALE: By saying she'll call her pregnant neighbor, the mother demonstrates that she understands the implications of rubella. Fetal defects can occur during the first trimester of pregnancy if the pregnant woman contracts rubella. Aspirin shouldn't be given to young children because aspirin has been implicated in the development of Reye's syndrome. Tylenol should be used instead of aspirin. Rubella immunization isn't recommended for children until ages 12 to 15 months. Having the measles (rubeola) won't provide immunity for rubella.

A nurse provides home care instructions to the parents of a child with congestive heart failure regarding the procedure for the administration of digoxin (Lanoxin). Which statement, if made by a parent, indicates the need for further instruction? 1. "I will not mix the medication with food." 2. "If more than one dose is missed, I will call the health care provider." 3. "I will take my child's pulse before administering the medication." 4. "If my child vomits after medication administration, I will repeat the dose."

4. "If my child vomits after medication administration, I will repeat the dose." Rationale: The parents need to be instructed that, if the child vomits after the digoxin is administered, they are not to repeat the dose. Options 1, 2, and 3 are accurate instructions regarding the administration of this medication. Additionally, the parents should be instructed that if a dose is missed and it is not noticed until 4 hours later, the dose should not be administered.

A toddler is in the hospital. The parents tell the nurse they're concerned about the seriousness of the child's illness. Which response to the parents is most appropriate? 1. "Please try not to worry. Your child will be fine." 2. "If you look around, you'll see other children who are much sicker." 3. "What seems to concern you about your child being hospitalized?" 4. "It must be difficult for you when your child is ill and hospitalized."

4. "It must be difficult for you when your child is ill and hospitalized." RATIONALE: Expressing concern is the most appropriate response because it acknowledges the parents' feelings. False reassurance, such as telling parents not to worry, isn't helpful because it doesn't acknowledge their feelings. Encouraging parents to look at how ill other children are also isn't helpful because the focus of the parents is on their own child. Asking what the concern is merely reinforces the parents' concern without addressing it.

A nurse is providing instructions to a mother of a child with cystic fibrosis regarding the immunization schedule for the child. Which statement should the nurse make to the mother? 1. "The immunization schedule will need to be altered." 2. "The child should not receive any hepatitis vaccines." 3. "The child will receive all of the immunizations except for the polio series." 4. "The child will receive the recommended basic series of immunizations along with a yearly influenza vaccination."

4. "The child will receive the recommended basic series of immunizations along with a yearly influenza vaccination." Rationale: It is essential that children with cystic fibrosis be adequately protected from communicable diseases by immunization. It is recommended that in addition to the basic series of immunizations, children with cystic fibrosis also should receive yearly influenza vaccines.

A nurse is preparing a child, age 4, for cardiac catheterization. Which explanation of the procedure is appropriate? 1. "Don't worry. It won't hurt." 2. "The test usually takes an hour." 3. "You must sleep the whole time that the test is being done." 4. "The special medicine will feel warm when it's put in the tubing."

4. "The special medicine will feel warm when it's put in the tubing." RATIONALE: To prepare a 4-year-old child without increasing anxiety, the nurse should provide concrete information in small amounts about nonthreatening aspects of the procedure. Therefore, saying the special medicine will feel warm is most appropriate. Saying that it won't hurt may prevent the child from trusting the nurse in the future. Explaining the time needed for the procedure wouldn't provide sufficient information. Stating that the child will need to sleep isn't true and could provoke anxiety.

A mother of a child with cystic fibrosis asks the nurse when the postural drainage should be performed. The mother states that the child eats meals at 8:00 AM, 12 noon, and at 6:00 PM The nurse tells the mother that the postural drainage should be performed at: 1. 8:00 AM, 2:00 PM, and 6:00 PM 2. 9:00 AM, 1:00 PM and 6:00 PM 3. 8:00 AM, 12:00 noon, and 6:00 PM 4. 10:00 AM, 2:00 PM and 8:00 PM

4. 10:00 AM, 2:00 PM and 8:00 PM Rationale: Respiratory treatments should be performed at least 1 hour before meals or 2 hours after meals to prevent vomiting. In some children with cystic fibrosis, treatments are prescribed every 2 hours, particularly if infection is present. It is also important to perform these treatments before bedtime to clear airways and facilitate rest.

A nurse is caring for a child recently diagnosed with cerebral palsy. The parents of the child ask the nurse about the disorder. The nurse bases the response to the parents on the understanding that cerebral palsy is: 1. An infectious disease of the central nervous system 2. An inflammation of the brain as a result of a viral illness 3. A congenital condition that results in moderate to severe retardation 4. A chronic disability characterized by a difficulty in controlling the muscles

4. A chronic disability characterized by a difficulty in controlling the muscles Rationale: Cerebral palsy is a chronic disability characterized by difficulty in controlling the muscles as a result of an abnormality in the extrapyramidal or pyramidal motor system. Meningitis is an infectious process of the central nervous system. Encephalitis is an inflammation of the brain that occurs as a result of viral illness or central nervous system infection. Down syndrome is an example of a congenital condition that results in moderate to severe retardation.

Which assessment should alert a nurse that a hospitalized 7-year-old child is at high risk for a severe asthma exacerbation? 1. Oxygen saturation of 95% 2. Mild work of breath 3. Intercostal or substernal retractions 4. A history of steroid-dependent asthma

4. A history of steroid-dependent asthma RATIONALE: The child's history of steroid-dependent asthma is a contributing factor to making him at high risk for a severe exacerbation. The nurse must treat the situation as a severe exacerbation regardless of the severity of the current episode. Decreased oxygen saturation, cyanosis, retractions, and increase (not mild) work of breathing are all assessments of an asthma exacerbation, not risk factors for it. These findings should be treated with oxygen, nebulized respiratory treatments, and steroids. However, if a significant history of high-risk factors is absent, the episode can be treated without hospitalization and followed up with the pediatrician.

A nurse is monitoring the daily weight of an infant with congestive heart failure (CHF). Which of the following alerts the nurse to suspect fluid accumulation and thus to the need to notify the registered nurse? 1. Bradypnea 2. Diaphoresis 3. Decreased blood pressure (BP) 4. A weight gain of 1 lb in 1 day

4. A weight gain of 1 lb in 1 day Rationale: A weight gain of 0.5 kg (1 lb) in 1 day is a result of the accumulation of fluid. The nurse should monitor the urine output, monitor for evidence of facial or peripheral edema, check the lung sounds, and report the weight gain. Tachypnea and an increased BP would occur with fluid accumulation. Diaphoresis is a sign of CHF, but it is not specific to fluid accumulation, and it usually occurs with exertional activities.

A child with rubeola (measles) is being admitted to the hospital. When preparing for the admission of the child, which precautions should be implemented? 1. PPE 2. Contact 3. Droplet 4. Airborne

4. Airborne Rationale: Rubeola is transmitted via airborne particles or direct contact with infectious droplets. Respiratory precautions are required, and a mask is worn by those who come in contact with the child. Gowns and gloves are not indicated. Articles that are contaminated should be bagged and labeled. Options 1, 2, and 3 are not indicated for rubeola.

Laboratory results for a child with a congenital heart defect with decreased pulmonary blood flow reveal an elevated hemoglobin (Hb) level, hematocrit (HCT), and red blood cell (RBC) count. These data suggest which condition? 1. Anemia 2. Dehydration 3. Jaundice 4. Compensation for hypoxia

4. Compensation for hypoxia RATIONALE: A congenital heart defect with decreased pulmonary blood flow alters blood flow through the heart and lungs, resulting in hypoxia. To compensate, the body increases the oxygen-carrying capacity of RBCs by increasing RBC production, which causes the Hb level and Hct to rise. In anemia, the Hb level and Hct typically decrease. Altered electrolyte levels and other laboratory values are better indicators of dehydration. An elevated Hb level and HCT aren't associated with jaundice.

A 10-month-old infant is admitted to the facility with dehydration and metabolic acidosis. What is the most common cause of dehydration and acidosis in infants? 1. Early introduction of solid foods 2. Inadequate perianal hygiene 3. Tachypnea 4. Diarrhea

4. Diarrhea RATIONALE: Diarrhea is the most common cause of dehydration and acidosis in infants. Early introduction of solid foods may cause loose stools but not dehydration or acidosis. Poor perianal hygiene may cause diaper dermatitis. Tachypnea is a sign — not a cause — of acidosis.

A nurse checks the vital signs of an infant with a respiratory infection and notes that the respiratory rate is 50 breaths per minute. Which action is appropriate? 1. Notify the registered nurse. 2. Administer oxygen. 3. Recheck the respiratory rate in 15 minutes. 4. Document the findings.

4. Document the findings. Rationale: The normal respiratory rate in an infant is 30 to 60 breaths per minute. The nurse would document the findings.

A nurse is caring for a 10-year-old child with cystic fibrosis. The child's parents tell the nurse that they're having difficulty coping with their child's disease. Which action should the nurse take? 1. Tell the parents they should be glad their child has lived this long. 2. Point out to the parents ways in which they might have done things differently. 3. Counsel the parents on not having any more children because they could also have cystic fibrosis. 4. Encourage the parents to allow their child to follow as normal a childhood as possible.

4. Encourage the parents to allow their child to follow as normal a childhood as possible. RATIONALE: The nurse should encourage the parents to treat their child as much like a normal child as possible. The nurse should avoid being critical of how parents handle their child's condition. Children with cystic fibrosis can live productive lives well into adulthood, so telling the parents they're lucky their child has lived this long not only is rude, it's inappropriate. Although each child the couple has has a 25% chance of having cystic fibrosis, it isn't appropriate for the nurse to counsel the parents. If they express uncertainty about having more children, the nurse should refer them to their physician or a genetic counselor.

A nurse reviews the record of a 3-week-old infant and notes that the health care provider has documented a diagnosis of suspected Hirschsprung's disease. The nurse understands that which of the following symptoms led the mother to seek health care for the infant? 1. Diarrhea 2. Projectile vomiting 3. The regurgitation of feedings 4. Foul-smelling, ribbon-like stools

4. Foul-smelling, ribbon-like stools Rationale: Chronic constipation that begins during the first month of life and that results in foul-smelling, ribbon-like or pellet-like stools is a clinical manifestation of Hirschsprung's disease. The delayed passage or absence of meconium stool during the neonatal period is a characteristic sign. Bowel obstruction (especially during the neonatal period), abdominal pain and distention, and failure to thrive are also clinical manifestations. Options 1, 2, and 3 are incorrect.

A mother brings her preschool child to the emergency department after the child ingested an unknown quantity of acetaminophen. Which treatment will the physician probably order? 1. Administration of a dose of ipecac syrup 2. Insertion of a nasogastric tube and administration of an antacid 3. I.V. infusion of normal saline solution 4. Gastric lavage and administration of activated charcoal

4. Gastric lavage and administration of activated charcoal RATIONALE: The physician will probably order gastric lavage or activated charcoal administration. Ipecac syrup is no longer recommended and an antacid isn't an effective treatment for poisoning. Infusing normal saline solution I.V. may be helpful in treating dehydration caused by vomiting, but in itself isn't effective in eliminating the poisonous substance.

A 2-month-old infant hasn't received any immunizations. Which immunizations should the nurse prepare to administer? 1. Measles, mumps, rubella (MMR); diphtheria, tetanus toxoids, and acellular pertussis (DTaP); and hepatitis B (HepB) 2. Polio (IPV), DTaP, MMR 3. Varicella, Haemophilus influenzae type b (HIB), IPV, and DTaP 4. HIB, DTaP, HepB, IPV, and pneumococcal conjugate vaccine (PCV)

4. HIB, DTaP, HepB, IPV, and pneumococcal conjugate vaccine (PCV) RATIONALE: The current immunizations recommended for a 2-month-old who hasn't received any immunizations are HIB, DTaP, HepB, PCV, and IPV. The first immunizations for MMR and varicella are recommended when a child is age 12 months.

A mother is discontinuing breast-feeding after 5 months. What should the nurse advise the mother to include in her infant's diet? 1. Iron-rich formula and baby food 2. Whole milk and baby food 3. Skim milk and baby food 4. Iron-rich formula alone

4. Iron-rich formula alone RATIONALE: The American Academy of Pediatrics recommends iron-rich formula for 5-month-old infants and cautions against giving infants solid food — even baby food — until age 6 months. The Academy doesn't recommend whole milk before age 12 months or skim milk before age 2 years.

A nurse is assessing an infant for signs of increased intracranial pressure (ICP). What is the earliest sign of increased ICP in an infant? 1. Vomiting 2. Papilledema 3. Vital sign changes 4. Irritability

4. Irritability RATIONALE: An infant with increased ICP is commonly fussy, irritable, and restless at first as a result of a headache cause by the ICP. Vomiting occurs later. Papilledema is a late sign of increased ICP that may not be evident. Changes in vital signs occur later; pressure on the brainstem slows pulse and respiration.

The primary goal to be included in the plan of care for a child who has cerebral palsy is to: 1. Eliminate the cause of the disease. 2. Improve muscle control and coordination. 3. Prevent the occurrence of emotional disturbances. 4. Maximize the child's assets and minimize the limitations.

4. Maximize the child's assets and minimize the limitations Rationale: The goal of managing the child with cerebral palsy is early recognition and intervention to maximize the child's abilities. The cause of the disease cannot be eliminated. It is best to minimize emotional disturbances, if possible, but not to prevent them because it is healthy for the child to express emotions. Improvement of muscle control and coordination is a component of the plan, but the primary goal is to maximize the child's assets and minimize the limitations caused by the disease.

What should a nurse do to ensure a safe hospital environment for a toddler? 1. Place the child in a youth bed. 2. Move stacking toys out of reach. 3. Pad the crib rails. 4. Move the equipment out of reach.

4. Move the equipment out of reach. RATIONALE: Moving the equipment out of reach ensures a safe environment because toddlers are curious and may try to play with items within their reach. Toddlers in a strange hospital environment still need the security of a crib. Stacking toys don't need to be moved out of reach because they don't present a safety hazard and are appropriate for this age-group. Padded crib rails are necessary only if seizure activity is present.

A nurse prepares to administer a Vitamin K injection to a newborn, and the mtoher asks the nurse why her infant needs the injection. The best response by the nurse would be: 1. Your newborn needs Vitamin K to develop immunity. 2. The Vitamin K will protect your newborn from being jaundiced. 3. Newborns have sterile bowels, and Vitamin K promotes the growth of bacteria in the bowel. 4. Newborns are deficient in Vitamin K, and this injection prevents your newborn from bleeding.

4. Newborns are deficient in Vitamin K, and this injection prevents your newborn from bleeding.

A nurse is collecting data about a child who has been admitted to the hospital with a diagnosis of seizures. The nurse checks for causes of the seizure activity by: 1. Testing the child's urine for specific gravity 2. Asking the child what happens during a seizure 3. Obtaining a family history of psychiatric illness 4. Obtaining a history regarding factors that may occur before the seizure activity

4. Obtaining a history regarding factors that may occur before the seizure activity Rationale: Fever and infections increase the body's metabolic rate. This can cause seizure activity among children who are less than 5-years-old. Dehydration and electrolyte imbalance can also contribute to the occurrence of a seizure. Falls can cause head injuries, which would increase intracranial pressure or cerebral edema. Some medications could cause seizures. Specific gravity would not be a reliable test, because it varies, depending on the existing condition. Psychiatric illness has no impact on seizure occurrence or cause. Children do not remember what happened during the seizure itself.

A nurse is assisting with data collection from an infant who has been diagnosed with hydrocephalus. If the infant's level of consciousness diminishes, a priority intervention is: 1. Taking the apical pulse 2. Taking the blood pressure 3. Testing the urine for protein 4. Palpating the anterior fontanel

4. Palpating the anterior fontanel Rationale: A full or bulging anterior fontanel indicates an increase in cerebrospinal fluid collection in the cerebral ventricle. Apical pulse and blood pressure changes and proteinuria are not specifically associated with increasing cerebrospinal fluid in the brain tissue in an infant.

A toddler is admitted to the facility for treatment of a severe respiratory infection. The child's recent history includes fatty stools and failure to gain weight steadily. The physician diagnoses cystic fibrosis. By the time of the child's discharge, the child's parents must be able to perform which task independently? 1. Allergy-proofing the home 2. Maintaining the child in an oxygen tent 3. Maintaining the child on a fat-free diet 4. Performing postural drainage

4. Performing postural drainage RATIONALE: The child with cystic fibrosis is at risk for frequent respiratory infections secondary to increased viscosity of mucus gland secretions. To help prevent respiratory infections, caregivers must perform postural drainage several times daily to loosen and drain secretions. Because exocrine gland dysfunction, not an allergic response, causes bronchial obstruction in cystic fibrosis, allergy-proofing the home isn't necessary. Oxygen therapy may be indicated, but only during acute disease episodes. Also, such therapy must be supervised closely; home oxygen therapy is inappropriate because chronic hypoxemia poses the risk of oxygen toxicity. If steatorrhea can't be controlled, the child should reduce, but not eliminate, dietary fat intake.

A nurse is assigned to care for an infant with tetralogy of Fallot. The mother of the infant calls the nurse to the room because the infant suddenly seems to be having difficulty breathing. The nurse enters the room and notes that the infant is experiencing a hypercyanotic episode. The initial nursing action is to: 1. Call a code. 2. Contact the respiratory therapy department. 3. Place the infant in a prone position. 4. Place the infant in a knee-chest position.

4. Place the infant in a knee-chest position. Rationale: If a hypercyanotic episode occurs, the infant is placed in a knee-chest position. The nurse would contact the registered nurse, who would then contact the health care provider. The knee-chest position is thought to increase pulmonary blood flow by increasing systemic vascular resistance. This position also improves systemic arterial oxygen saturation by decreasing venous return so that smaller amounts of highly saturated blood reach the heart. Toddlers and children squat to obtain this position and relieve chronic hypoxia.

A child with cerebral palsy (CP) is working to achieve maximum potential for locomotion, self-care, and socialization in school. The nurse would work with the child to meet these goals by: 1. Keeping the child in a special education classroom with other children with similar disabilities 2. Laying the child in the supine position with a 30-degree elevation of the head to facilitate feeding 3. Removing ankle-foot orthoses and braces once the child arrives at school 4. Placing the child on a wheeled scooter board

4. Placing the child on a wheeled scooter board Rationale: Option 4 provides the child with maximum potential in locomotion, self-care, and socialization. The child can move around independently on the abdomen anywhere the child wants to go and can interact with others as desired. Orthoses must be used all the time to aid locomotion (option 3). Option 1 does not provide for maximum socialization and normalization, rather, children with CP need to be mainstreamed as much as cognitively able. Not all children with CP are intellectually challenged. Option 2 does not provide for normalization in self-care. Just as children without CP sit up and use assistive devices when eating, so should children with CP.

When assessing a child with bronchiolitis, which finding does the nurse expect? 1. Clubbed fingers 2. Barrel chest 3. Barking cough and stridor 4. Productive cough

4. Productive cough RATIONALE: Bronchiolitis causes a productive cough. Clubbed fingers and a barrel chest are more likely in a client with chronic respiratory problems. A barking cough is associated with croup.

A nurse teaches a mother how to provide adequate nutrition for her toddler, who has cerebral palsy. Which observation indicates that teaching has been effective? 1. The toddler stays neat while eating. 2. The toddler finishes the meal within a specified period of time. 3. The child lies down to rest after eating. 4. The child eats finger foods by himself.

4. The child eats finger foods by himself. RATIONALE: The child eating finger foods by himself indicates effective teaching because a child with cerebral palsy should be encouraged to be as independent as possible. Finger foods allow the toddler to feed himself. Because spasticity affects coordinated chewing and swallowing as well as the ability to bring food to the mouth, it's difficult for the child with cerebral palsy to eat neatly. In terms of a specified period of time, the child with cerebral palsy may require more time to bring food to the mouth; thus, chewing and swallowing shouldn't be rushed. A child shouldn't lie down to rest after eating because doing so may cause the child to vomit from a hyperactive gag reflex. Therefore, the child should remain in an upright position after eating to prevent aspiration and choking.

The nurse is monitoring a client in labor. The nurse suspects umbilical cord compression if which is noted on the external monitor tracing during a contraction? 1. Variability 2. Acceleration 3. Early decelerations 4. Variable decelerations

4. Variable decelerations Variable decelerations occur if the umbilical cord is compressed, reducing the blood flow between the placenta and the fetus.

What immunization is contraindicated if the patient's history indicates an allergy to eggs? 1. Polio 2. Hepatitis B 3. Respiratory syncytial virus 4. Varicella

4. Varicella An allergy to eggs should be reported before receiving MMR, varicella, or influenza vaccines.

A 4-month-old infant is taken to the pediatrician by his parents because they're concerned about his frequent respiratory infections, poor feeding habits, frequent vomiting, and colic. The physician notes that the infant has failed to gain expected weight and recommends that the infant have a sweat test performed to detect possible cystic fibrosis. To prepare the parents for the test, the nurse should explain that: 1. the baby will need to fast before the test. 2. a sample of blood will be necessary. 3. a low-sodium diet is necessary for 24 hours before the test. 4. a low-intensity, painless electrical current is applied to the skin.

4. a low-intensity, painless electrical current is applied to the skin. RATIONALE: Because cystic fibrosis clients have elevated levels of sodium and chloride in their sweat, a sweat test is performed to confirm this disorder. The nurse should explain to the parents that after pilocarpine (a cholinergic medication that induces sweating) is applied to a gauze pad and placed on the arm, a low-intensity, painless electrical current is applied for several minutes. The arm is then washed off, and a filter paper is placed over the site with forceps to collect the sweat. Elevated levels of sodium and chloride are diagnostic of cystic fibrosis. No fasting is necessary before this test and no blood sample is required. A low-sodium diet isn't required before the test.

A nurse is assessing an 8-month-old child for signs of neurologic deficit and increased intracranial pressure (ICP). These signs include: 1. a depressed fontanel. 2. slurred speech. 3. tachycardia. 4. an altered level of consciousness.

4. an altered level of consciousness. RATIONALE: One sign of neurologic deficit in an 8-month-old child includes a decreased or altered level of consciousness. The fontanel would bulge — not depress — if he had increased ICP. Slurred speech isn't a sign of increased ICP in an infant because the child isn't able to speak at this age. However, a change in cry may be noted. Bradycardia — not tachycardia — is a sign of increased ICP.

A physician orders digoxin elixir for a toddler with heart failure. Immediately before administering this drug, the nurse must check the toddler's: 1. serum sodium level. 2. urine output. 3. weight. 4. apical pulse.

4. apical pulse. RATIONALE: Because digoxin may reduce the heart rate and heart failure may cause a pulse deficit, the nurse should measure the toddler's apical pulse before administering the drug to prevent further slowing of the heart rate. The serum sodium level doesn't affect digoxin's action. For a child with heart failure, the nurse should check urine output and measure weight regularly, but not necessarily just before digoxin administration.

A nurse is preparing to administer the first dose of tobramycin (Nebcin) to an adolescent with cystic fibrosis. The order is for 3 mg/kg I.M. daily in three divided doses. The client weighs 95 lb. How many milligrams should the nurse administer per dose? Record your answer using one decimal place. Answer: milligrams

43.2 milligrams RATIONALE: To perform this dosage calculation, the nurse should first convert the client's weight to kilograms using this formula: 1 kg/2.2 lb = X kg/95 lb 2.2X = 95 X = 43.2 kg Then, she should calculate the client's daily dose using this formula: 43.2 kg × 3 mg/kg = 129.6 mg Lastly, the nurse should calculate the divided dose: 129.6 mg ÷3 doses = 43.2 mg/dose

A nurse is preparing a dose of amoxicillin for a 3-year-old child with acute otitis media. The child weighs 33 lb. The dosage ordered is 50 mg/kg/day in divided doses every 8 hours. The concentration of the drug is 250 mg/5 ml. How many milliliters should the nurse administer? Record your answer using a whole number. Answer: milliliters

5 milliliters RATIONALE: To calculate the child's weight in kilograms, the nurse should use the following formula: 2.2 lb/1 kg = 33 lb/X kg X = 33 ÷ 2.2 X = 15 kg. Next, the nurse should calculate the daily dosage for the child: 50 mg/kg/day × 15 kg = 750 mg/day. To determine divided daily dosage, the nurse should know that "every 8 hours" means 3 times per day. So, she should perform that calculation in this way: Total daily dosage ÷ 3 times per day = Divided daily dosage 750 mg/day ÷ 3 = 250 mg The drug's concentration is 250 mg/5 ml, so nurse should administer 5 ml.

The perinatal nurse is caring for a woman in the immediate postbirth period. Assessment reveals that the woman is experiencing profuse bleeding. The most likely etiology for the bleeding is: A. Uterine atony B. Uterine inversion C. Vaginal hematoma D. Vaginal laceration

A A. Correct: Uterine atony is marked hypotonia of the uterus. It is the leading cause of postpartum hemorrhage. B. Incorrect: Uterine inversion may lead to hemorrhage, but it is not the most likely source of this client's bleeding. Furthermore, if the woman was experiencing a uterine inversion, it would be evidenced by the presence of a large, red, rounded mass protruding from the introitus. C. Incorrect: A vaginal hematoma may be associated with hemorrhage. However, the most likely clinical finding would be pain, not the presence of profuse bleeding. D. Incorrect: A vaginal laceration may cause hemorrhage, but it is more likely that profuse bleeding would result from uterine atony. A vaginal laceration should be suspected if vaginal bleeding continues in the presence of a firm, contracted uterine fundus.

With regard to the process of augmentation of labor, the nurse should be aware that it: a. Is part of the active management of labor that is instituted when the labor process is unsatisfactory. b. Relies on more invasive methods when oxytocin and amniotomy have failed. c. Is a modern management term to cover up the negative connotations of forceps-assisted birth. d. Uses vacuum cups.

A Augmentation is part of the active management of labor that stimulates uterine contractions after labor has started but is not progressing satisfactorily. Augmentation uses amniotomy and oxytocin infusion, as well as some gentler, noninvasive methods. Forceps-assisted births and vacuum-assisted births are appropriately used at the end of labor and are not part of augmentation.

A pregnant woman's amniotic membranes rupture. Prolapsed umbilical cord is suspected. What intervention would be the top priority? a. Placing the woman in the knee-chest position b. Covering the cord in sterile gauze soaked in saline c. Preparing the woman for a cesarean birth d. Starting oxygen by face mask

A The woman is assisted into a position (e.g., modified Sims position, Trendelenburg position, or the knee-chest position, on all fours) in which gravity keeps the pressure of the presenting part off the cord. Although covering the cord in sterile gauze soaked saline, preparing the woman for a cesarean, and starting oxygen by face mark are appropriate nursing interventions in the event of a prolapsed cord, the intervention of top priority would be positioning the mother to relieve cord compression.

Select all the patients below who are at risk for developing placenta previa:* A. A 37 year old woman who is pregnant with her 7th child. B. A 28 year old pregnant female with chronic hypertension. C. A 25 year old female who is 36 weeks pregnant that has experienced trauma to abdomen. D. A 20 year old pregnant female who is a cocaine user.

A and D. Risk factors for developing placenta previa include: Maternal age >35 years old, multiples (twins etc.), already had a baby, drug use: cocaine or smoking, surgery to the uterus that will leave scarring: fibroid removal, c-section etc.

A child diagnosed with cerebral palsy has damage to the cerebellum. Which type of movement problems will the healthcare provider anticipate? Choose all answers that apply: A) Clumsiness B) Unsteady gait C) Hyperreflexia D) Twisting movements E) Problems with balance

A, B, E

A child diagnosed with cerebral palsy has recurring absence seizures. Which of the following are characteristic of this type of seizure? Choose all answers that apply: A) Lip smacking B) Bicycling movements C) Blank stare D) Generalized contractions

A, C

A nurse assists with admitting a child with a diagnosis of Kawasaki disease. When obtaining the child's medical history, which clinical manifestations is likely to be reported? Select all that apply. A. "Strawberry" tongue B. Rhinorrhea C. Nonexudative conjunctivitis D. Irritability

A, C, D Children can have irritiabilty, red rash, a "strawberry" colored tounge, desquamation of the fingers, swollen hands, fever, red eyes without drainage.

Select all the signs and symptoms associated with placenta previa:* A. Painless bright red bleeding B. Concealed bleeding C. Hard, tender uterus D. Normal fetal heart rate E. Abnormal fetal position F. Rigid abdomen

A, D, and E. These are all sign and symptoms of placenta previa. The other options are associated abruptio placentae.

A nurse is reviewing a health care provider's prescription for a child who was just admitted to the hospital with a diagnosis of Kawasaki disease. Which prescription should the nurse anticipate being part of the treatment plan? A. Immune globulin B. Heparin infusion C. Vitamin K injection D. Digoxin

A. Immune globulin Rationale: Intravenous immune globulin (IVIG) is administered to the child with Kawasaki disease to decrease the incidence of coronary artery lesions and aneurysms and to decrease fever and inflammation. Options 2, 3, and 4 are not components of the treatment plan for this disease.

You're performing a head-to-toe assessment on a patient admitted with abruptio placentae. Which of the following assessment findings would you immediately report to the physician?* A. Oozing around the IV site B. Tender uterus C. Hard abdomen D. Vaginal bleeding

A. Oozing around the IV site can indicate the patient is entering into DIC (disseminated intravascular coagulation) because clotting levels have been depleted. Therefore, the MD should be notified. Option B, C, and D are findings found in this condition, but Option A is a SEVERE complication that can develop from it.

A nurse is preparing for the admission of an infant with a diagnosis of bronchiolitis caused by the respiratory syncytial virus (RSV). Choose the interventions that would be included in the plan of care. Select all that apply. A. Place the infant in a private room. B. Place the infant in a room near the nurses' station. C. Ensure that the infant's head is in a flexed position. D. Wear a mask at all times when in contact with the infant. E. Place the child in a tent that delivers warm, humidified air. F. Position the infant side-lying, with the head lower than the chest.

A. Place the infant in a private room. B. Place the infant in a room near the nurses' station. Rationale: The infant with RSV should be isolated in a private room or in a room with another child with RSV. The infant should be placed in a room near the nurses' station for close observation. The infant should be positioned with the head and chest at a 30- to 40-degree angle and the neck slightly extended to maintain an open airway and to decrease pressure on the diaphragm. Cool, humidified oxygen is delivered to relieve dyspnea, hypoxemia, and insensible water loss from tachypnea. Contact precautions (wearing gloves and a gown) reduce the nosocomial transmission of RSV.

A newborn's one minute APGAR score is 8. Which of the following nursing interventions will you provide to this newborn?* A. Routine post-delivery care B. Full resuscitation assistance is needed and reassess APGAR score C. Continue to monitor and reassess the APGAR score in 10 minutes D. Some resuscitation assistance such as oxygen

A. Scoring Interventions are as follows: 7-10: no interventions, baby doing good just needs routine post-delivery care, 4-6: some resuscitation assistance required like oxygen, suction.... stimulate the baby, rub baby's back, 0-3: needs full resuscitation *Remember the APGAR scoring is performed at 1 minute and 5 minutes after birth and reassessed at 10 minutes (5 minutes later) after birth if the score is 6 or less

You're performing a routine assessment on a mother post-delivery. The uterus is soft and displaced to the left of the umbilicus. What is your next nursing action?* A. Perform fundal massage and assist the patient to the bathroom. B. Continue to monitor the mother. This is a normal finding post-delivery. C. Notify the physician. D. Administer PRN dose of Pitocin as ordered by the physician.

A. This is not a normal finding. The fundus of the uterus should be firm, mid-line, and near or at the umbilicus. If the fundus is soft, boggy, and displaced, the nurse should perform fundal massage and assist the patient to the bathroom to void. A full bladder can cause the fundus to become displaced and soft/boggy.

A nurse is preparing to administer digoxin to an infant with congestive heart failure (CHF). Before administering the medication, the nurse double-checks the dose, counts the apical heart rate for 1 full minute, and obtains a rate of 88 beats per minute. Based on this finding, which of the following is the appropriate nursing action? A. Withhold the medication. B. Administer the medication. C. Double-check the apical heart rate and administer the medication. D. Check the blood pressure and respirations and administer the medication.

A. Withhold the medication. Rationale: Safety in dosing is achieved by double-checking the dose and counting the apical heart rate for 1 full minute. If the heart rate is less than 100 beats per minute in an infant, the nurse would withhold the dose and notify the registered nurse and health care provider.

You're assessing the one minute APGAR score of a newborn baby. On assessment, you note the following about your newborn patient: heart rate 101, cyanotic body and extremities, no response to stimulation, no flexion of extremities, and strong cry. What is your patient's APGAR score?* A. APGAR 4 B. APGAR 6 C. APGAR 3 D. APGAR 2

A: APGAR 4.....A: 0, P: 2, G, 0, A: 0, R: 2

Which pelvic shape is ideal for a vaginal birth? __________________

ANS: Gynecoid The gynecoid pelvis is the most common; major gynecoid pelvic features are present in 50% of all women. Anthropoid and android features are less common, and platypelloid pelvic features are the least common.

The volume of amniotic fluid is an important factor in assessing fetal well-being. Oligohydramnios (an amniotic fluid volume of less than 300 ml) is associated with what kind of fetal anomalies? a. Renal b. Cardiac c. Gastrointestinal d. Neurologic

ANS: A An amniotic fluid volume of less than 300 ml (oligohydramnios) is associated with fetal renal anomalies. The amniotic fluid volume has no bearing on the fetal cardiovascular system. Gastrointestinal anomalies are associated with hydramnios, or an amniotic fluid volume greater than 2 L. The amniotic fluid volume has no bearing on the fetal neurologic system.

A pregnant woman tells her nurse that she is worried about the blotchy, brownish coloring over her cheeks, nose, and forehead. The nurse can reassure her that this is a normal condition related to hormonal change, commonly called the mask of pregnancy or, scientifically: a. Chloasma b. Linea nigra c. Striae gravidarum d. Palmar erythema

ANS: A Chloasma, the mask of pregnancy, usually fades after birth. Linea nigra is a pigmented line that runs vertically up the abdomen. Striae gravidarum are also known as stretch marks. Palmar erythema is signified by pinkish red blotches on the hands.

A nurse caring for a laboring woman is cognizant that early decelerations are caused by: a. Altered fetal cerebral blood flow b. Umbilical cord compression c. Uteroplacental insufficiency d. Spontaneous rupture of membranes

ANS: A Early decelerations are the fetus's response to fetal head compression. These are considered benign and interventions are not necessary. Variable decelerations are associated with umbilical cord compression. Late decelerations are associated with uteroplacental insufficiency. Spontaneous rupture of membranes has no bearing on the fetal heart rate (FHR) unless the umbilical cord prolapses, which would result in variable or prolonged bradycardia.

Which description of the four stages of labor is correct for both definition and duration? a. First stage: onset of regular uterine contractions to full dilation; less than 1 hour to 20 hours b. Second stage: full effacement to 4 to 5 cm; visible presenting part; 1 to 2 hours c. Third stage: active pushing to birth; 20 minutes (multiparous women), 50 minutes (first-timer) d. Fourth stage: delivery of the placenta to recovery; 30 minutes to 1 hour

ANS: A Full dilation may occur in less than 1 hour, but in first-time pregnancies it can take up to 20 hours. The second stage extends from full dilation to birth and takes an average of 20 to 50 minutes, although 2 hours is still considered normal. The third stage extends from birth to expulsion of the placenta and usually takes a few minutes. The fourth stage begins after expulsion of the placenta and lasts until homeostasis is reestablished (about 2 hours).

The most critical nursing action in caring for the newborn immediately after birth is: a. Keeping the airway clear b. Fostering parent-newborn attachment c. Drying the newborn and wrapping the infant in a blanket d. Administering eye drops and vitamin K

ANS: A The care given immediately after the birth focuses on assessing and stabilizing the newborn. Although fostering parent-newborn attachment is an important task for the nurse, it is not the most critical nursing action in caring for the newborn immediately after birth. The care given immediately after birth focuses on assessing and stabilizing the newborn. The nursing activities are (in order of importance) to maintain a patent airway, support respiratory effort, and prevent cold stress by drying the newborn and covering with a warmed blanket or placing the newborn under a radiant warmer. After the newborn has been stabilized, the nurse assesses the newborn's physical condition, weighs and measures the newborn, administers prophylactic eye ointment and a vitamin K injection, affixes an identification bracelet, wraps the newborn in warm blankets, and then gives the partner or the mother the infant.

When assessing a woman in the first stage of labor, the nurse recognizes that the most conclusive sign that uterine contractions are effective is: a. Dilation of the cervix b. Descent of the fetus c. Rupture of the amniotic membranes d. Increase in bloody show

ANS: A The vaginal examination reveals whether the woman is in true labor. Cervical change, especially dilation, in the presence of adequate labor indicates that the woman is in true labor. Descent of the fetus, or engagement, may occur before labor. Rupture of membranes may occur with or without the presence of labor. Bloody show may indicate slow, progressive cervical change (e.g., effacement) in both true and false labor.

When planning care for a laboring woman whose membranes have ruptured, the nurse recognizes that the woman's risk for _____ has increased. a. Intrauterine infection b. Hemorrhage c. Precipitous labor d. Supine hypotension

ANS: A When the membranes rupture, microorganisms from the vagina can ascend into the amniotic sac, causing chorioamnionitis and placentitis. Rupture of membranes (ROM) is not associated with fetal or maternal bleeding. Although ROM may increase the intensity of contractions and facilitate active labor, it does not result in precipitous labor. ROM has no correlation with supine hypotension.

The diagnosis of pregnancy is based on which positive signs of pregnancy? Choose all that apply. a. Identification of fetal heartbeat b. Palpation of fetal outline c. Visualization of the fetus d. Verification of fetal movement e. Positive human chorionic gonadotropin (hCG) test

ANS: A, C, D Identification of fetal heartbeat, visualization of the fetus, and verification of fetal movement are all positive, objective signs of pregnancy. Palpation of fetal outline and positive hCG test are probable signs of pregnancy. A tumor also can be palpated. Medication and tumors may lead to false-positive results on pregnancy tests.

The musculoskeletal system adapts to the changes that occur during pregnancy. A woman can expect to experience what change? a. Her center of gravity will shift backward. b. She will have increased lordosis. c. She will have increased abdominal muscle tone. d. She will notice decreased mobility of her pelvic joints.

ANS: B An increase in the normal lumbosacral curve (lordosis) develops, and a compensatory curvature in the cervicodorsal region develops to help her maintain her balance. The center of gravity shifts forward. She will have decreased muscle tone. She will notice increased mobility of her pelvic joints.

A newly married couple plans to use natural family planning. It is important for them to know how long an ovum can live after ovulation. The nurse knows that teaching is effective when the couple responds that an ovum is considered fertile for: a. 6 to 8 hours b. 24 hours c. 2 to 3 days d. 1 week

ANS: B Ova are considered fertile for about 24 hours after ovulation. Ova are considered fertile for much longer than 6 to 8 hours. Most remain fertile for 24 hours. Ova do not remain fertile for 2 to 3 days. If unfertilized by a sperm, the ovum degenerates and is reabsorbed. Ova do not remain viable for 1 week. After 24 hours the ovum degenerates and is reabsorbed.

The nurse expects to administer an oxytocic (e.g., Pitocin, Methergine) to a woman after expulsion of her placenta to: a. Relieve pain b. Stimulate uterine contraction c. Prevent infection d. Facilitate rest and relaxation

ANS: B Oxytocics stimulate uterine contractions, which reduce blood loss after the third stage of labor. Oxytocics are not used to treat pain. Oxytocics do not prevent infection. Oxytocics do not facilitate rest and relaxation.

Probable signs of pregnancy are: a. Determined by ultrasound b. Observed by the health care provider c. Reported by the client d. Diagnostic tests

ANS: B Probable signs are those detected through trained examination. Fetal visualization is a positive sign of pregnancy. Presumptive signs are those reported by the client. The term diagnostic tests is open for interpretation. To actually diagnose pregnancy, one would have to see positive signs of pregnancy.

During labor a fetus with an average fetal heart rate (FHR) of 135 beats/min over a 10-minute period is considered to have: a. Bradycardia b. A normal baseline heart rate c. Tachycardia d. Hypoxia

ANS: B The baseline heart rate is measured over 10 minutes; a normal range is 110 to 160 beats/min. Bradycardia is a FHR less than 110 beats/min for 10 minutes or longer. Tachycardia is a FHR more than 160 beats/min for 10 minutes or longer. Hypoxia is an inadequate supply of oxygen; no indication of this condition exists with a baseline heart rate in the normal range.

The nurse providing care for the laboring woman understands that variable fetal heart rate (FHR) decelerations are caused by: a. Altered fetal cerebral blood flow b. Umbilical cord compression c. Uteroplacental insufficiency d. Fetal hypoxemia

ANS: B Variable decelerations can occur any time during the uterine contracting phase and are caused by compression of the umbilical cord. Altered fetal cerebral blood flow results in early decelerations in the FHR. Uteroplacental insufficiency results in late decelerations in the FHR. Fetal hypoxemia results in tachycardia initially, then bradycardia if hypoxia continues.

The nurse knows that the second stage of labor, the descent phase, has begun when: a. The amniotic membranes rupture b. The cervix cannot be felt during a vaginal examination c. The woman experiences a strong urge to bear down d. The presenting part is below the ischial spines

ANS: C During the descent phase of the second stage of labor, the woman may experience an increase in the urge to bear down. Rupture of membranes (ROM) has no significance in determining the stage of labor. The second stage of labor begins with full cervical dilation. Many women may have an urge to bear down when the presenting part is below the level of the ischial spines. This can occur during the first stage of labor, as early as 5 cm dilation.

Some pregnant clients may complain of changes in their voice and impaired hearing. The nurse can tell these clients that these are common reactions to: a. A decreased estrogen level b. Displacement of the diaphragm, resulting in thoracic breathing c. Congestion and swelling, which occur because the upper respiratory tract has become more vascular d. Increased blood volume

ANS: C Estrogen levels increase, causing the upper respiratory tract to become more vascular; this produces swelling and congestion in the nose and ears and therefore voice changes and impaired hearing. Estrogen levels increase, not decrease. The diaphragm is displaced. However, the key is that estrogen levels increase, causing the upper respiratory tract to become more vascular; this produces swelling and congestion in the nose and ears and therefore voice changes and impaired hearing. The volume of blood is increased. However, the key here is that estrogen levels increase, causing the upper respiratory tract to become more vascular; this produces swelling and congestion in the nose and ears and therefore voice changes and impaired hearing.

A nurse caring for a woman in labor understands that maternal hypotension can result in: a. Early decelerations b. Fetal arrhythmias c. Uteroplacental insufficiency d. Spontaneous rupture of membranes

ANS: C Low maternal blood pressure reduces placental blood flow during uterine contractions, resulting in fetal hypoxemia. Maternal hypotension does not result in early decelerations. Maternal hypotension is not associated with fetal arrhythmias. Spontaneous rupture of membranes is not a result of maternal hypotension.

A nurse providing care to a pregnant woman should know that all are normal gastrointestinal changes in pregnancy except: a. Ptyalism b. Pyrosis c. Pica d. Decreased peristalsis

ANS: C Pica (a desire to eat nonfood substances) is an indication of iron deficiency and should be evaluated. Ptyalism (excessive salivation) is a normal finding. Pyrosis (heartburn) is a normal finding. Decreased peristalsis is a normal finding.

A 27-year-old pregnant woman had a preconceptual body mass index (BMI) of 19. The nurse knows that this woman's total recommended weight gain during pregnancy should be at least: a. 20 kg (44 lb) b. 16 kg (35 lb) c. 12.5 kg (27.5 lb) d. 10 kg (22 lb)

ANS: C This woman has a normal BMI and should gain 11.5 to 16 kg during pregnancy. A weight gain of 20 kg (44 lb) is unhealthy for most women. This woman has a normal BMI and should gain 11.5 to 16 kg during pregnancy. A weight gain of 16 kg (35 lb) is the high end of the range of weight this woman should gain in her pregnancy. A weight gain of 10 kg (22 lb) is appropriate for an obese woman. This woman has a normal BMI, which indicates that her weight is average.

A woman arrives at the clinic for a pregnancy test. Her last menstrual period (LMP) was February 14, 2011. Her expected date of birth (EDB) is: a. September 17, 2011 b. November 7, 2011 c. November 21, 2011 d. December 17, 2011

ANS: C Using Nägele's rule, the EDB is calculated by subtracting 3 months from the month of the LMP and adding 7 days + 1 year to the day of the LMP. Therefore, with an LMP of February 14, 2011, her due date is November 21, 2011. September 17, 2011, is too short a period to complete a normal pregnancy. Using Nägele's rule, an EDB of November 7, 2011 is 2 weeks early. December 17, 2011, is almost a month past the correct EDB.

A woman has come to the clinic for preconception counseling because she wants to start trying to get pregnant in 3 months. She can expect the following advice: a. "Discontinue all contraception now." b. "Lose weight so that you can gain more during pregnancy." c. "You may take any medications you have been taking regularly." d. "Make sure you include adequate folic acid in your diet."

ANS: D A healthy diet before conception is the best way to ensure that adequate nutrients are available for the developing fetus. A woman's folate or folic acid intake is of particular concern in the periconception period. Neural tube defects are more common in infants of women with a poor folic acid intake. Depending on the type of contraception used, it may not be appropriate to discontinue all contraception at this time. Advising the client to lose weight now so that she can gain more during pregnancy is not appropriate advice. Depending on the type of medications the woman is taking, it may not be appropriate for her to continue taking them regularly.

The most basic information a maternity nurse should have concerning conception is: a. Ova are considered fertile 48 to 72 hours after ovulation b. Sperm remain viable in the woman's reproductive system for an average of 12 to 24 hours c. Conception is achieved when a sperm successfully penetrates the membrane surrounding the ovum d. Implantation in the endometrium occurs 6 to 10 days after conception

ANS: D After implantation, the endometrium is called the decidua. Ova are considered fertile for about 24 hours after ovulation. Sperm remain viable in the woman's reproductive system for an average of 2 to 3 days. Penetration of the ovum by the sperm is called fertilization. Conception occurs when the zygote, the first cell of the new individual, is formed.

Human chorionic gonadotropin (hCG) is an important biochemical marker for pregnancy and therefore the basis for many tests. A maternity nurse should be aware that: a. hCG can be detected as early as 2½ weeks after conception b. The hCG level increases gradually and uniformly throughout pregnancy c. Much lower than normal increases in the level of hCG may indicate a postdate pregnancy d. A higher than normal level of hCG may indicate an ectopic pregnancy or Down syndrome

ANS: D Higher levels also could be a sign of multiple gestation. hCG can be detected as early as 7 to 10 days after conception. The hCG level fluctuates during pregnancy, peaking, declining, stabilizing, and then increasing again. Abnormally slow increases may indicate impending miscarriage.

In order to reassure and educate pregnant clients about changes in the cervix, vagina, and position of the fetus, nurses should be aware that: a. Because of a number of changes in the cervix, abnormal Papanicolaou (Pap) tests are much easier to evaluate b. Quickening is a technique of palpating the fetus to engage it in passive movement c. The deepening color of the vaginal mucosa and cervix (Chadwick sign) usually appears in the second trimester or later as the vagina prepares to stretch during labor d. Increased vascularity of the vagina increases sensitivity and may lead to a high degree of arousal, especially in the second trimester

ANS: D Increased sensitivity and an increased interest in sex sometimes go together. This frequently occurs during the second trimester. These cervical changes make evaluation of abnormal Pap tests more difficult. Quickening is the first recognition of fetal movements by the mother. Ballottement is a technique used to palpate the fetus. The Chadwick sign appears from the sixth to eighth weeks.

Which fetal heart rate (FHR) finding concerns the nurse during labor? a. Accelerations with fetal movement b. Early decelerations c. An average FHR of 126 beats/min d. Late decelerations

ANS: D Late decelerations are caused by uteroplacental insufficiency and are associated with fetal hypoxemia. They are considered ominous if persistent and uncorrected. Accelerations in the FHR are an indication of fetal well-being. Early decelerations in the FHR are associated with head compression as the fetus descends into the maternal pelvic outlet; they generally are not a concern during normal labor. An FHR finding of 126 beats/min is normal and not a concern.

For what reason is breastfeeding contraindicated? a. Hepatitis B b. Everted nipples c. History of breast cancer 3 years ago d. Human immunodeficiency virus (HIV) positive

ANS: D Women who are HIV positive are discouraged from breastfeeding. Although hepatitis B antigen has not been shown to be transmitted through breast milk, as an added precaution infants born to HBsAg-positive women should receive the hepatitis B vaccine and immune globulin immediately after birth. Everted nipples are functional for breastfeeding. Newly diagnosed breast cancer is a contraindication to breastfeeding.

A nurse is preparing to administer the first dose of tobramycin (Nebcin) to an adolescent with cystic fibrosis. The order is for 3 mg/kg I.V. daily in three divided doses. The client weighs 95 lb. How many milligrams should the nurse administer per dose? Record your answer using one decimal place.

Answer: 43.2 milligrams RATIONALE: To perform this dosage calculation, the nurse should first convert the client's weight to kilograms using this formula: 1 kg/2.2 lb = X kg/95 lb 2.2X = 95 X = 43.2 kg Then, she should calculate the client's daily dose using this formula: 43.2 kg × 3 mg/kg = 129.6 mg Lastly, the nurse should calculate the divided dose: 129.6 mg ÷3 doses = 43.2 mg/dose

Which of the following instructions should be included in the nurse's teaching regarding oral contraceptives? a. Weight gain should be reported to the physician. b. An alternate method of birth control is needed when taking antibiotics. c. If the client misses one or more pills, two pills should be taken per day for 1 week. d. Changes in the menstrual flow should be reported to the physician.

Answer: B When the client is taking oral contraceptives and begins antibiotics, another method of birth control should be used. Antibiotics decrease the effectiveness of oral contraceptives. Approximately 5-10 pounds of weight gain is not unusual, so answer A is incorrect. If the client misses a birth control pill, she should be instructed to take the pill as soon as she remembers the pill. Answer C is incorrect. If she misses two, she should take two; if she misses more than two, she should take the missed pills but use another method of birth control for the remainder of the cycle. Answer D is incorrect because changes in menstrual flow are expected in clients using oral contraceptives. Often these clients have lighter menses.

A adult female patient is using the rhythm (calendar-basal body temperature) method of family planning. In this method, the unsafe period for sexual intercourse is indicated by; a. Return preovulatory basal body temperature b. Basal body temperature increase of 0.1 degrees to 0.2 degrees on the 2nd or 3rd day of cycle c. 3 full days of elevated basal body temperature and clear, thin cervical mucus d. Breast tenderness and mittelschmerz

Answer: C Ovulation (the period when pregnancy can occur) is accompanied by a basal body temperature increase of 0.7 degrees F to 0.8 degrees F and clear, thin cervical mucus. A return to the preovulatory body temperature indicates a safe period for sexual intercourse. A slight rise in basal temperature early in the cycle is not significant. Breast tenderness and mittelschmerz are not reliable indicators of ovulation.

Which would indicate concealed hemorrhage in abruptio placentae? a. Bradycardia b. Hard boardlike abdomen c. Decrease in fundal height d. Decrease in abdominal pain

B ~ Concealed hemorrhage occurs when the edges of the placenta do not separate. The formation of a hematoma behind the placenta and subsequent infiltration of the blood into the uterine muscle results in a very firm, boardlike abdomen. The client will have shock symptoms that include tachycardia. The fundal height will increase as bleeding occurs. Abdominal pain may increase.

Which maternal condition always necessitates birth by cesarean section? a. Partial abruptio placentae b. Total placenta previa c. Ectopic pregnancy d. Eclampsia

B ~ In total placenta previa, the placenta completely covers the cervical os. The fetus would die if a vaginal birth occurred. If the client has stable vital signs and the fetus is alive, a vaginal birth can be attempted. If the fetus has died, a vaginal birth is preferred. The most common ectopic pregnancy is a tubal pregnancy, which is usually detected and treated in the first trimester. Labor can be safely induced if the eclampsia is under control.

The nurse is preparing a health teaching program for parents of toddlers and preschoolers and plans to include information about prevention of accidental poisonings. It is most important for the nurse to include which instruction? A) Tell children they should not taste anything but food. B) Store all toxic agents and medicines in locked cabinets. C) Provide special play areas in the house and restrict play in other areas. D) Punish children if they open cabinets that contain household chemicals.

B) The only reliable way to prevent poisonings in young children is to make them inaccessible, B. Teaching children not to taste is important, A, but ineffective for young children. C and D, will not control a child's curiosity.

A 36 year old woman, who is 38 weeks pregnant, reports having dark red bleeding. The patient experienced abruptio placentae with her last pregnancy at 29 weeks. What other signs and symptoms can present with abruptio placentae? Select all that apply:* A. Decrease in fundal height B. Hard abdomen C. Fetal distress D. Abnormal fetal position E. Tender uterus

B, C, and E. Option A is wrong because there may be an INCREASE in fundal height (not decrease) due to concealed bleeding. Option D is wrong because this tends to occur in placenta previa because the placenta attaches too low in the uterus at the cervical opening.

Your patient who is 34 weeks pregnant is diagnosed with total placenta previa. The patient is A positive. What nursing interventions below will you include in the patient's care? Select all that apply:* A. Routine vaginal examinations B. Monitoring vital signs C. Administer RhoGAM per MD order D. Assess internal fetal monitoring E. Placing patient on side-lying position F. Monitoring pad count G. Monitoring CBC and clotting levels

B, E, F, and G. Option A is WRONG because vaginal exams are avoided to prevent causing damage to the placenta presenting at the cervical opening. Option C is WRONG because the patient is A positive and does NOT need RhoGAM, which is for patients who are RH negative. Option D is WRONG because external monitoring should be used NOT internal, which can damage the placenta at the cervical opening.

A nurse has reinforced home care instructions to the mother of a child who is being discharged after cardiac surgery. Which statement, if made by the mother, indicates the need for further instructions? A. "A balance of rest and exercise is important." B. "I can apply lotion or powder to the incision if it is itchy." C. "Activities during which the child could fall need to be avoided for 2 to 4 weeks." D. "Large crowds of people need to be avoided for at least 2 weeks after this surgery."

B. "I can apply lotion or powder to the incision if it is itchy." Rationale: The mother should be instructed that lotions and powders should not be applied to the incision site because these items can affect the skin integrity and the healing process. Options 1, 3, and 4 are accurate instructions regarding home care after cardiac surgery.

Which statement made by an expectant mother demonstrates understanding of the significant risks of home delivery? A. "I know I will have access to the technology that monitors my well-being." B. "I know that there will be a delay in emergency care if there is a complication." C. "The physician will only come to my home if I have a complication." D. "The midwife can perform most emergency procedures at home."

B. "I know that there will be a delay in emergency care if there is a complication." Mothers will not have access to technology at home. Most physicians will not come to the home for medical care. Most emergency procedures can only be performed in the hospital per standard of care. It is important that this mother understands that there will be a delay, creating significant risk, if there is a complication.

A child recently diagnosed with asthma has a prescription to use an inhaled medication with a spacer. The nurse evaluates the child has correct understanding of the use of an inhaler with a spacer when the child states which of these? A. "I don't have to wait a minute between the two puffs if I use a spacer." B. "If the spacer makes a whistling sound, I am breathing in too fast." C. "I should rinse my mouth and then swallow the water to get all of the medicine." D. "I should shake the canister when I want to see whether it is empty." E. "I should hold my breath for at least ten seconds after inhaling the medication."

B. "If the spacer makes a whistling sound, I am breathing in too fast." E. "I should hold my breath for at least ten seconds after inhaling the medication."

You're assessing the five minute APGAR score of a newborn baby. On assessment, you note the following about your newborn patient: pink body and hands with cyanotic feet, heart rate 109, grimace to stimulation, flaccid, and irregular cry. What is your patient's APGAR score?* A. APGAR 8 B. APGAR 5 C. APGAR 6 D. APGAR 3

B. APGAR 5:...A: 1, P: 2, G, 1, A: 0, R: 1

A nurse prepares to administer a pancreatic enzyme powder to the child with cystic fibrosis (CF). Which of the following food items will the nurse mix with the medication? A. Tapioca B. Applesauce C. Hot oatmeal D. Mashed potatoes

B. Applesauce Rationale: Pancreatic enzyme powders are not to be mixed with hot foods or foods containing tapioca or other starches. Enzyme powder should be mixed with non-fat, non-protein foods such as applesauce. Pancreatic enzymes are inactivated by heat and are partially degraded by gastric acids.

A nurse is caring for an infant with a diagnosis of tetralogy of Fallot. The infant suddenly becomes cyanotic and the oxygen saturation reading drops to 60%. Choose the interventions that the nurse should perform. Select all that apply. A. Call a code blue. B. Notify the HCP C. Place the infant in a prone position. D. Prepare to administer 100% oxygen by face mask.

B. Notify the HCP. D. Prepare to administer 100% oxygen by face mask. Rationale: The child who is cyanotic with oxygen saturations dropping to 60% is having a hypercyanotic episode. Hypercyanotic episodes often occur among infants with tetralogy of Fallot, and they may occur among infants whose heart defect includes the obstruction of pulmonary blood flow and communication between the ventricles. If a hypercyanotic episode occurs, the infant is placed in a knee-chest position immediately. The HCP notified. The knee-chest position improves systemic arterial oxygen saturation by decreasing venous return so that smaller amounts of highly saturated blood reach the heart. Toddlers and children squat to get into this position and relieve chronic hypoxia. There is no reason to call a code blue unless respirations cease. Additional interventions include administering 100% oxygen by face mask and intravenous fluids, as prescribed.

Regarding the scenario in the question above, when would you reassess the APGAR?* A. 2 minutes B. 10 minutes C. 5 minutes D. No reassessment of the APGAR score is needed.

B. The APGAR score is performed at 1 minute and 5 minutes after birth and reassessed at 10 minutes (5 minutes later) after birth, IF the score is 6 or less.

What statement is FALSE about the transition phase of stage 1?* A. The mother may experience intense pain, irritation, nausea, and deep concentration. B. The transition phase is the longest phase of stage 1 and contractions are very intense and long in duration. C. The cervix will dilate from 8 to 10 cm. D. The transition phase ends and progresses to stage 2 of labor when the cervix has dilated to 10 cm.

B. This option is false. It should say: The transition phase is the SHORTEST (not longest) phase of stage 1 and contractions are very intense and long in duration.

The clinic nurse is performing a prenatal assessment on a pregnant client at risk for preeclampsia. Which clinical sign is not included as a symptom of preeclampsia? a. Edema b. Proteinuria c. Glucosuria d. Hypertension

C ~ Glucose into the urine is not one of the three classic symptoms of preeclampsia. The first sign noted by the pregnant client is rapid weight gain & edema of the hands & face. Proteinuria usually develops later than the edema & hypertension. The first indication of preeclampsia is usually an increase in the maternal blood pressure.

Which finding on a prenatal visit at 10 weeks might suggest a hydatidiform mole? a. Blood pressure of 120/80 mm Hg b. Complaint of frequent mild nausea c. Fundal height measurement of 18 cm d. History of bright red spotting for 1 day weeks ago

C ~ The uterus in a hydatidiform molar pregnancy is often larger than would be expected on the basis of the duration of the pregnancy. A client with a molar pregnancy may have early-onset, pregnancy-induced hypertension. Nausea increases in a molar pregnancy because of the increased production of human chorionic gonadotropin (hCG). The history of bleeding is normally described as being of a brownish color.

A nurse who is working in the Poison Control Center receives several telephone calls from parents whose children have ingested possible poisons. The nurse should recommend inducing vomiting for which child? A) 8-month-old who ate 4 to 6 ibuprofen tablets. B) 3-year-old who drank an unknown amount of charcoal lighter fluid. C) 16-month old who drank 2 ounces of acetaminophen (Tylenol) elixir. D) 2-year-old who ate a handful of automatic dishwasher detergent

C) Emesis should be induced for the child who drank the large dose of acetaminophen, Tylenol, elixir, C, because this medication is hepatotoxic. Vomiting is contraindicated for: children under 1 year of age, A, petroleum distillates, B, such as charcoal lighter fluid, and corrosives, D, such as dishwasher detergents.

A 39-year-old woman is undergoing prenatal screening for Down syndrome. Which of the following diagnostic tests gives the most accurate information about the probability of Down syndrome? A) Ultrasound B) Amniotic fluid index C) Amniocentesis D) Serum alpha fetoprotein (AFP)

C) amniocentesis

A client had a previous cesarean birth. What are the criteria in order to try having a vaginal birth during the second pregnancy? Select all that apply. A. A history of postpartum hemorrhage B. A previous classical vertical incision C. Clinically adequate pelvis D. Previous low transverse incision E. No history of uterine rupture

C, D, E (A vaginal birth is possible after a previous caesarean delivery if the pelvis is found to be adequate to provide room for childbirth. A previous low transverse incision poses less risk of rupture and a vaginal delivery may be possible. A client with no history of uterine rupture would have less risk of uterine rupture during the vaginal delivery. A history of postpartum hemorrhage may not affect the risk associated with a second vaginal delivery in women with a history of first caesarean delivery. A previous vertical incision on the uterus increases the risk of uterine rupture.)

After receiving education on the correct use of emergency drug therapy for asthma, which statement by the adolescent indicates a correct understanding of the nurse's instructions? A."All asthma drugs help everybody breathe better." B. "I must carry my emergency inhaler when activity is anticipated." C. "I must have my emergency inhaler with me at all times." D. "Preventive drugs can stop an attack."

C. "I must have my emergency inhaler with me at all times."

A woman, who is 22 weeks pregnant, has a routine ultrasound performed. The ultrasound shows that the placenta is located at the edge of the cervical opening. As the nurse you know that which statement is FALSE about this finding:* A. This is known as marginal placenta previa. B. The placenta may move upward as the pregnancy progresses and needs to be re-evaluated with another ultrasound at about 32 weeks gestation. C. The patient will need to have a c-section and cannot deliver vaginally. D. The woman should report any bleeding immediately to the doctor.

C. All the other options are CORRECT. Option C is FALSE. This is a type of placenta previa called marginal (or low-lying). There is a chance the woman can delivery vaginally, but if the placenta was completely over the cervix or partially covering it a c-section would be required. At the 20 week ultrasound the location of the placenta is detected. The location will be re-evaluated at about 32 weeks. If a placenta is found to be low lying there is a chance the placenta will move upward (away from the cervix) as the uterus grows to accommodate the baby.

Which statement is TRUE regarding abruptio placenta?* A. This condition occurs due to an abnormal attachment of the placenta in the uterus near or over the cervical opening. B. A marginal abruptio placenta occurs when the placenta is located near the edge of the cervical opening. C. Nursing interventions for this condition includes measuring the fundal height. D. Fetal distress is not common in this condition as it is in placenta previa.

C. All the other options are INCORRECT.

The mother has delivered the placenta. You note that the shiny surface of the placenta was delivered first. What delivery mechanism is this known as AND is this the maternal or baby's surface of the placenta?* A. Duncan mechanism, maternal B. Schultze mechanism, maternal C. Schultze mechanism, baby D. Duncan mechanism, baby

C. Remember "SHINY" Schultze. This is the side from the baby. Try to remember the baby is shiny and new so it is the SHINY Schultze side. The Schultze mechanism is where the baby's surface is delivered first. Duncan mechanism is where the maternal side is delivered first. Remember "DULL/DIRTY" Duncan. This side will be dull/dirty, red, and rough and is the side from the mother. Try to remember the mother is dirty from labor and is in rough shape.

A newborn's five minute APGAR score is 5. Which of the following nursing interventions will you provide to this newborn?* A. Routine post-delivery care B. Continue to monitor and reassess the APGAR score in 10 minutes. C. Some resuscitation assistance such as oxygen and rubbing baby's back and reassess APGAR score. D. Full resuscitation assistance is needed and reassess APGAR score.

C. Scoring Interventions are as follows: 7-10: no interventions, baby doing good just needs routine post-delivery care, 4-6: some resuscitation assistance required like oxygen, suction.... stimulate the baby, rub baby's back, 0-3: needs full resuscitation

During stage 3 of labor, you note a gush of blood and that the uterus changes shape from an oval shape to globular shape. This indicates? A. Postpartum hemorrhage B. Imminent delivery of the baby C. Signs of placental separation D. Answers B and C

C. Signs that the placenta is about to be delivered: Umbilical cord starts to lengthen, Trickling/gush of blood, and uterus changes from an oval shape to globular.

In stage 1 of labor, during the active phase, the cervix dilates?* A. 1-3 cm B. 7-10 cm C. 4-7 cm D. 8-10 cm

C: 4-7 cm

You're assessing the one minute APGAR score of a newborn baby. On assessment, you note the following about your newborn patient: heart rate 130, pink body and hands with cyanotic feet, weak cry, flexion of the arms and legs, active movement and crying when stimulated. What is your patient's APGAR score?* A. APGAR 9 B. APGAR 10 C. APGAR 8 D. APGAR 5

C: APGAR 8.....A: 1, P: 2, G, 2, A: 2, R: 1

A parent brings a 4-month-old infant to the clinic for the second in the routine immunization series. The nurse should prepared for administration of which of the following vaccinations? 1.DTap, Hib, polio 2. DTap, polio, MMR 3. DTap, polio, varicella 4. Td, hepatitis, MMR

Correct answer: 1 Rationale: During the second round of routine immunizations at the 4-month pediatric visit, the nurse should prepare to administer DTaP, Hib, and polio vaccines.

How are pinworms diagnosed? 1.) seeing the worm in the stool 2.) a blood antigen level 3.) A "Scotch tape test" in the early morning 4.) a stool laboratory examination obtained at the hour of sleep

Correct answer: 3 Rationale: A special pinworm diagnostic tape or paddle, or a tongue blade covered with cellophane tape with the sticky side out, may be placed against the anal region to obtain pinworm eggs (the "Scotch tape test"). This is done early in the morning, before the child has a bowel movement, bathes, or scratches the area with the fingers. The tape is put on a glass slide and examined under a microscope. The eggs are typical of pinworms.

The DTap immunization is administered: 1. orally 2. subcutaneously 3. intramuscularly 4. intravenously

Correct answer: 3 Rationale: The DTaP immunization is administered intramuscularly.

Which menu selections are best for a child diagnosed with celiac disease? 1.) pizza and chocolate cake 2.) spaghetti and blueberry muffin 3.) chicken sandwich on whole-wheat bread 4.) corn tortilla and fresh fruit

Correct answer: 4 Rationale: Celiac disease is also known as gluten enteropathy and sprue, and it is the leading malabsorption problem in children. Repeated exposure to gluten damages the villi in the mucous membranes of the intestine. Gluten is found in wheat, barley, oats, and rye. Foods containing these are restricted for those with the disease.

Distinct phases of a grand-mal epileptic seizure include (SATA): 1. aura 2. agitation 3. tonic/clonic movements 4. postictal lethargy

Correct answers: 1, 3, 4 Rationale: The most common generalized seizure is the tonic-clonic or grand mal. A grand mal epilepsy has three distinct phases: an aura (subjective sensation), a tonic-clonic seizure, and postictal lethargy or a short period of sleep. A person experiencing this type of seizure will present with a sudden cry (or aura), fall, and rigidity, followed by muscle jerking; shallow, irregular breathing; and possible loss of bladder or bowel control. It usually lasts seconds to minutes, followed by some confusion, a period of sleep, and then a return to full consciousness.

The nurse is monitoring a pregnant client after amniotomy. Which observation would indicate a likelihood of umbilical cord compression? A. The fetal heart rate (FHR) confirms tachycardia. B. The client's vaginal drainage has a foul-smell. C. The client has maternal chills frequently. D. The fetal heart rate (FHR) has variable decelerations.

D (Amniotomy is performed in a pregnant client in order to rupture the membranes artificially. After the procedure, the nurse should closely monitor the FHR. Reduced FHR and variable decelerations in FHR indicate that the client's umbilical cord is compressed. The nurse should immediately inform the primary health care provider of the client's condition. Tachycardia or increased FHR are common manifestations observed after amniotomy. Tachycardia does not require immediate clinical action. Maternal chills and foul-smelling vaginal discharge after amniotomy indicate infection of the ruptured membranes. However, this would not be a reason to expect umbilical cord compression.)

Nurses should be aware that the induction of labor: a. Can be achieved by external and internal version techniques. b. Is also known as a trial of labor (TOL). c. Is almost always done for medical reasons. d. Is rated for viability by a Bishop score.

D A high score (above 6) is predictive of successful labor induction because the cervix has ripened or softened in preparation for labor. Version is turning of the fetus to a better position by a physician for an easier or safer birth. A trial of labor is the observance of a woman and her fetus for several hours of active labor to assess the safety of vaginal birth. Two thirds of cases of induced labor are elective and are not done for medical reasons.

The nurse practicing in a labor setting knows that the woman most at risk for uterine rupture is: a. A gravida 3 who has had two low-segment transverse cesarean births. b. A gravida 2 who had a low-segment vertical incision for delivery of a 10-pound infant. c. A gravida 5 who had two vaginal births and two cesarean births. d. A gravida 4 who has had all cesarean births.

D The risk of uterine rupture increases for the patient who has had multiple prior births with no vaginal births. As the number of prior uterine incisions increases, so does the risk for uterine rupture. Low-segment transverse cesarean scars do not predispose the patient to uterine rupture.

A nurse provides home care instructions to the parents of a child with congestive heart failure regarding the procedure for the administration of digoxin. Which statement, if made by a parent, indicates the need for further instruction? A. "I will not mix the medication with food." B. "If more than one dose is missed, I will call the health care provider." C. "I will take my child's pulse before administering the medication." D. "If my child vomits after medication administration, I will repeat the dose."

D. "If my child vomits after medication administration, I will repeat the dose." Rationale: The parents need to be instructed that, if the child vomits after the digoxin is administered, they are not to repeat the dose. Options 1, 2, and 3 are accurate instructions regarding the administration of this medication. Additionally, the parents should be instructed that if a dose is missed and it is not noticed until 4 hours later, the dose should not be administered.

A nurse is instructing the mother of a child with cystic fibrosis (CF) about the appropriate dietary measures. Which of the following meals best illustrates the most appropriate diet for a client with cystic fibrosis? A. A veggie salad and a caramel apple B. A strawberry jelly sandwich and pretzels C. A plate of nachos and cheese and a cupcake D. A piece of fried chicken and a loaded baked potato

D. A piece of fried chicken and a loaded baked potato Rationale: Children with CF are managed with a high-calorie, high-protein diet. Pancreatic enzyme replacement therapy is undertaken, and fat-soluble vitamin supplements are administered. Fats are not restricted unless steatorrhea cannot be controlled by increased levels of pancreatic enzymes. A piece of fried chicken and a loaded baked potato provides a high-calorie and high-protein meal that includes fat.

A nurse is monitoring the daily weight of an infant with congestive heart failure (CHF). Which of the following alerts the nurse to suspect fluid accumulation and thus to the need to notify the registered nurse? A. Bradypnea B. Diaphoresis C. Decreased blood pressure (BP) D. A weight gain of 1 lb in 1 day

D. A weight gain of 1 lb in 1 day A weight gain of 0.5 kg (1 lb) in 1 day is a result of the accumulation of fluid. The nurse should monitor the urine output, monitor for evidence of facial or peripheral edema, check the lung sounds, and report the weight gain. Tachypnea and an increased BP would occur with fluid accumulation. Diaphoresis is a sign of CHF, but it is not specific to fluid accumulation, and it usually occurs with exertional activities.

A nurse caring for an infant with bronchiolitis is monitoring for signs of dehydration. The nurse monitors which of the following as the reliable method of determining fluid loss? A. Intake B. Output C. Skin turgor D. Body weight

D. Body weight Rationale: Body weight is the most reliable method of measurement of body fluid loss or gain. One kilogram of weight change represents 1 L of fluid loss or gain. Although options A, B, and C may be used to determine fluid status, they are not the most reliable determinants.

Your laboring patient has transitioned to stage 2 of labor. What changes in the perineum indicate the birth of the baby is imminent?* A. Increase in meconium-stained fluid and retracting perineum B. Retracting perineum and anus with an increase of bloody show C. Rapid and intense contractions D. Bulging perineum and rectum with an increase in bloody show

D. Bulging perineum and rectum with an increase in bloody show (and presenting of the baby's head or other parts) are signs that the birth of the baby imminent.

After birth, where do you expect to assess fundal height?* A. At the xiphoid process B. 5 cm below the umbilicus C. 2 cm above the pubic symphysis D. At or near the umbilicus

D. It should be found at or near the umbilicus. It will decrease 1 cm a day and after 10 days post-delivery it can not be palpated.

At what point does preeclampsia become eclampsia? A.Onset of diplopia and headache B. Blood pressure of 150/100 mm Hg or above C. Presence of facial edema and proteinuria D. One or more generalized tonic-clonic seizures

D. One or more generalized tonic-clonic seizures

True or False: A normal fetal heart rate is between 100-120 bpm.* True False

FALSE. A normal fetal heart rate is between 110-160 bpm.

True or False: Stage 2 of labor begins with the delivery of the baby and ends with the delivery of the placenta.* True False

FALSE. Stage 2 begins with the full dilation of the cervix (10 cm) and ends with the full delivery of the baby. Stage 3 is the delivery of the placenta

True or False: Stage 4 of labor starts with the full delivery of the baby and ends with the full delivery of the placenta.* True False

FALSE. Stage 4 is 1-4 hours AFTER the delivery of the placenta. This statement describes stage 3.

What physical characteristic most likely indicates postmaturity? Abundant lanugo Peeling skin Abundant vernix caseosa Thin, transparent skin

Peeling skin The postterm infant is long and thin and looks as though weight has been lost. The skin is loose, especially about the thighs and buttocks. There is little lanugo or vernix caseosa. The skin is dry; it cracks, peels, and is almost like parchment in texture. The nails are long and may be stained with meconium. The infant has a thick head of hair and looks alert.

Which diagnostic test would be used first to evaluate male fertility? Endocrine test Semen analysis Ultrasound Testicular biopsy

Semen analysis is inexpensive and easy to obtain, so it is the first test to evaluate fertility. The others are done, but they are more costly or invasive.

Which patient history and physical information would contraindicate use of oral contraceptives? Select all that apply. Heavy menstrual bleeding Irregular bleeding Undiagnosed menstrual bleeding Positive pregnancy test One week postpartum and breastfeeding

Undiagnosed menstrual bleeding Positive pregnancy test One week postpartum and breastfeeding Women with undiagnosed bleeding disorders should not take oral contraceptives (BCP). A woman who has been evaluated for irregular or heavy bleeding is sometimes prescribed BCP to manage bleeding. A woman who thinks she may be pregnant or is lactating should not take BCP.

When developing a postoperative care plan for an infant scheduled for cleft lip repair, the nurse should assign highest priority to which intervention? 1. Comforting the child as quickly as possible 2. Maintaining the child in a prone position 3. Restraining the child's arms at all times, using elbow restraints 4. Avoiding disturbing any crusts that form on the suture line

1. Comforting the child as quickly as possible RATIONALE: After surgery to repair a cleft lip, the primary goal of nursing care is to maintain integrity of the operative site. Crying causes tension on the suture line, so comforting the child as quickly as possible is the highest nursing priority. Parents may help by cuddling and comforting the child. The prone position is contraindicated after surgery because rubbing on the sheet may disturb the suture line. Elbow restraints may cause agitation; if used to prevent the child from disturbing the suture line, they must be removed, one at a time, every 2 hours so that the child can exercise and the nurse can assess for skin irritation. Crusts forming on the suture line contribute to scarring and must be cleaned carefully.

What medication should be readily available when immunizations are administered? 1. Epinephrine 2. Amoxicillin 3. Mucomyst 4. Ativan

1. Epinephrine Epinephrine should be available in the unit where vaccines are administered, and the child should be observed for a minimum of 20 minutes before he or she leaves the area.

Twenty-four hours after birth, a neonate hasn't passed meconium. The nurse suspects which condition? 1. Hirschsprung's disease 2. Celiac disease 3. Intussusception 4. Abdominal wall defect

1. Hirschsprung's disease RATIONALE: Failure to pass meconium is an important diagnostic indicator for Hirschsprung's disease. Hirschsprung's disease is a potentially life-threatening congenital large-bowel disorder characterized by the absence or marked reduction of parasympathetic ganglion cells in a segment of the colorectal wall; narrowing impairs intestinal motility and causes severe, intractable constipation leading to partial or complete colonic obstruction. Celiac disease, intussusception, and abdominal wall defects aren't associated with failure to pass meconium.

A physician orders acetaminophen (Tylenol) elixir, 160 mg every 4 hours, for a 14-month-old child who weighs 20 lb (9.08 kg). This drug, supplied in a bottle labeled 160 mg/tsp, has a safe dosage of 10 mg/kg/dose. The nurse should administer how many milliliters? 1. None because this isn't a safe dose 2. 2.5 ml 3. 5 ml 4. 7.5 ml

1. None because this isn't a safe dose RATIONALE: For this client, the safe dose of this drug is 90.8 mg (9.08 kg × 10 mg/kg = 90.8 mg). This dose is equivalent to 2.8 ml. Therefore, the ordered dose isn't safe.

A licensed practical nurse (LPN) is bathing a neonate and notices small dark tufts of fine hair on the neonate's lower back. The LPN should take which best action? 1. Notify the registered nurse of the finding. 2. Assess for other associated anomalies and document carefully. 3. Tell the mother and father that this may indicate spina bifida. 4. Recognize that this is normal in the neonate and continue the bath.

1. Notify the registered nurse of the finding. Rationale: The legal role of the LPN is to practice under the supervision of the registered nurse. In this instance, the tuft of hair may be indicative of a spinal anomaly, and the registered nurse should be notified of the finding. It is inappropriate to discuss abnormal findings with the parents because this is the responsibility of the health care provider, if an anomaly is suspected or diagnosed. The LPN should take the priority intervention of notifying the registered nurse before documenting in the chart.

A nurse reinforces home-care instructions to the parents of a child with celiac disease. Which of the following food items would the nurse advise the parents to include in the child's diet? 1. Rice 2. Oatmeal 3. Rye toast 4. Wheat bread

1. Rice Rationale: Dietary management is the mainstay of treatment for celiac disease. All wheat, rye, barley, and oats should be eliminated from the diet and replaced with corn and rice. Vitamin supplements, especially fat-soluble vitamins and folate, may be required during the early period of treatment to correct deficiencies. These restrictions are likely to be lifelong, although small amounts of grains may be tolerated after the gastrointestinal ulcerations have healed. Rice, in its natural form, is gluten free. This includes white, brown, and wild rice.

The nurse in a newborn nursery is monitoring a preterm newborn for respiratory distress syndrome. Which assessment findings would alert the nurse to the possibility of this syndrome? 1. Tachypnea and retractions 2. Acrocyanosis and grunting 3. Hypotension and bradycardia 4. Presence of barrel chest and acrocyanosis

1. Tachypnea and retractions A newborn infant with respiratory distress syndrome may present with clinical signs of cyanosis, tachypnea or apnea, nasal flaring, chest wall retractions, or audible grunts.

A child is being discharged with proventil (Albuterol) nebulizer treatments. The nurse should instruct the parents to watch for: 1. tachycardia. 2. bradypnea. 3. urine retention. 4. constipation.

1. tachycardia. RATIONALE: Proventil is a beta-adrenergic blocker bronchodilator used to relieve bronchospasms associated with acute or chronic asthma or other obstructive airway diseases. Signs and symptoms of proventil toxicity that the nurse should instruct the parents to watch for include tachycardia, restlessness, nausea, vomiting, and dizziness. Unusually slow respirations, urine retention, and constipation aren't associated with proventil toxicity.

Sudden infant death syndrome (SIDS) is one of the most common causes of death in infants. At what age is SIDS most likely to occur? 1. 1 to 2 years 2. 1 week to 1 year, peaking at 2 to 4 months 3. 6 months to 1 year, peaking at 10 months 4. 6 to 8 weeks

2. 1 week to 1 year, peaking at 2 to 4 months RATIONALE: SIDS can occur anytime between ages 1 week and 1 year. The incidence peaks at ages 2 to 4 months.

Sudden infant death syndrome (SIDS) is one of the most common causes of death in infants. At what age is SIDS most likely to occur? 1. 1 to 2 years 2. 1 week to 1 year, peaking at 2 to 4 months 3. 6 months to 1 year, peaking at 10 months 4. 6 to 8 weeks

2. 1 week to 1 year, peaking at 2 to 4 months RATIONALE: SIDS can occur anytime between ages 1 week and 1 year. The incidence peaks at ages 2 to 4 months.

A mother of a child with cystic fibrosis asks the clinic nurse about the disease. The nurse tells the mother that it is: 1. Transmitted as an autosomal dominant trait 2. A chronic multisystem disorder affecting the exocrine glands 3. A disease that causes the formation of multiple cysts in the lungs 4. A disease that causes dilation of the passageways of many organs

2. A chronic multisystem disorder affecting the exocrine glands Rationale: Cystic fibrosis is a chronic multisystem disorder affecting the exocrine glands. The mucus produced by these glands (particularly those of the bronchioles, small intestine, and the pancreatic and bile ducts) is abnormally thick, causing obstruction of the small passageways of these organs. It is transmitted as an autosomal recessive trait.

Which toy is appropriate for a 3-year-old child? 1. A bicycle 2. A puzzle with large pieces 3. A pull toy 4. A computer game

2. A puzzle with large pieces RATIONALE: A puzzle is the most appropriate toy because, at age 3, children like to color, draw, and put together puzzles. A bicycle is appropriate for a 5- or 6-year-old child; a pull toy, for a toddler; and a computer game, for a school-age child.

The nurse is preforming an assessment of a client who is scheduled for a cesarean delivery. Which assessment finding would indicate the need to contact the health care provider? 1. Hemoglobin of 11g/dL 2. Fetal heart rate of 180 beats/minute 3. Maternal pulse rate of 85 beats/minute 4. White blood cell count of 12,000 cells/mm3

2. Fetal heart rate of 180 beats/minute

A nurse is caring for an infant with a diagnosis of tetralogy of Fallot. The infant suddenly becomes cyanotic and the oxygen saturation reading drops to 60%. Choose the interventions that the nurse should perform. Select all that apply. 1. Call a code blue. 2. Notify the registered nurse. 3. Place the infant in a prone position. 4. Prepare to administer intravenous fluids. 5. Prepare to administer 100% oxygen by face mask.

2. Notify the registered nurse. 5. Prepare to administer intravenous fluids. 6. Prepare to administer 100% oxygen by face mask. Rationale: The child who is cyanotic with oxygen saturations dropping to 60% is having a hypercyanotic episode. Hypercyanotic episodes often occur among infants with tetralogy of Fallot, and they may occur among infants whose heart defect includes the obstruction of pulmonary blood flow and communication between the ventricles. If a hypercyanotic episode occurs, the infant is placed in a knee-chest position immediately. The registered nurse is notified, who will then contact the health care provider. The knee-chest position improves systemic arterial oxygen saturation by decreasing venous return so that smaller amounts of highly saturated blood reach the heart. Toddlers and children squat to get into this position and relieve chronic hypoxia. There is no reason to call a code blue unless respirations cease. Additional interventions include administering 100% oxygen by face mask, morphine sulfate, and intravenous fluids, as prescribed.

A nurse is conducting an infant nutrition class for parents. Which foods are appropriate to introduce during the first year of life? Select all that apply. 1. Sliced beef 2. Pureed fruits 3. Whole milk 4. Rice cereal 5. Strained vegetables 6. Fruit juice

2. Pureed fruits 4. Rice cereal 5. Strained vegetables RATIONALE: The first food provided to a neonate is breast milk or formula. Between ages 4 and 6 months, rice cereal can be introduced, followed by pureed or strained fruits and vegetables, then strained, chopped or ground meat. Infants shouldn't be given whole milk until they are at least age 1. Fruit drinks provide no nutritional benefit and shouldn't be encouraged.

The nurse in the newborn nursery is preparing to feed a newborn the first feeding of sterile water. During the feeding, the newborn suddenly begins to cough, choke, and become cyanotic. Based on these symptoms, the nurse might suspect that the newborn has which of the following conditions? 1. Atrial septal defect 2. Tracheoesophageal fistula 3. Bronchopulmonary dysplasia 4. Respiratory distress syndrome

2. Tracheoesophageal fistula Rationale: The first feeding a newborn receives is sterile water to assess whether the newborn might have one of the tracheoesophageal (TE) conditions. Although sterile water is more easily absorbed and causes less aspiration than formula, the newborn with a suspected TE fistula condition will cough and choke during feedings. These symptoms are not associated with the conditions noted in options 1, 3, or 4.

The nurse is assessing a pregnant client in the second trimester of pregnancy who was admitted to the maternity unit with a suspected diagnosis of abruptio placentae. Which assessment finding should the nurse expect to note if the condition is present? 1. Soft abdomen 2. Uterine tenderness 3. Absence of abdominal pain 4. Painless, bright red vaginal bleeding

2. Uterine tenderness Abruptio placentae is the premature separation of the placenta from the uterine wall after the twentieth week of gestation and before the fetus is delivered. Abdominal pain is present along with uterine tenderness.

For a child who's admitted to the emergency department with an acute asthma attack, nursing assessment is most likely to reveal: 1. apneic periods. 2. expiratory wheezing. 3. inspiratory stridor. 4. fine crackles throughout.

2. expiratory wheezing. RATIONALE: Expiratory wheezing is common during an acute asthma attack and results from narrowing of the airway caused by edema. Acute asthma rarely causes apneic periods. Inspiratory stridor more commonly accompanies croup. The child may have some fine crackles but wheezing is much more common in an acute asthma attack.

A child, age 2, with a history of recurrent ear infections is brought to the clinic with a fever and irritability. To elicit the most pertinent information about the child's ear problems, the nurse should ask the parent: 1. "Does your child's ear hurt?" 2. "Does your child have any hearing problems?" 3. "Does your child tug at either ear?" 4. "Does anyone in your family have hearing problems?"

3. "Does your child tug at either ear?" RATIONALE: Although all of the options are appropriate questions to ask when assessing a young child's ear problems, questions about the child's behavior, such as "Does your child tug at either ear?" are most useful because a young child usually can't describe symptoms accurately.

An 8-month-old infant is admitted with a febrile seizure. The infant weighs 17 lb (7.7 kg). The physician orders ceftriaxone (Rocephin), 270 mg I.M. every 12 hours. (The safe dosage range is 50 to 75 mg/kg daily.) The pharmacy sends a vial containing 500 mg, to which the nurse adds 2 ml of preservative-free normal saline solution. The nurse should administer how many milliliters? 1. None because this isn't a safe dosage 2. 0.08 ml 3. 1.08 ml 4. 1.8 ml

3. 1.08 ml RATIONALE: Because the infant weighs 17 lb (7.7 kg), the safe dosage range is 385 to 578 mg daily. The ordered dosage, 540 mg daily, is safe. To calculate the amount to administer, the nurse may use the following fraction method: 500 mg/2 ml = 270 mg/X ml 500X = 270 × 2 500X = 540 X = 540/500 X = 1.08 ml

An 18-month-old boy is admitted to the pediatric unit with a diagnosis of celiac disease. What finding would the nurse expect in this child? 1. A concave abdomen 2. Bulges in the groin area 3. A protuberant abdomen 4. A palpable abdominal mass

3. A protuberant abdomen RATIONALE: The nurse would expect to find a protuberant abdomen caused by the presence of fat, bulky stools; undigested food; and flatus, which are associated with celiac disease. A concave abdomen, bulges in the groin area, and a palpable abdominal mass aren't associated with celiac disease.

A nurse is developing goals for a school-age child with a knowledge deficit related to the use of inhalers and peak flowmeters. The nurse identifies which of the following as an appropriate goal for this child? 1. Denies shortness of breath or difficulty breathing 2. Has regular respirations at a rate of 18 to 22 breaths per minute 3. Expresses feelings of mastery and competence with breathing devices 4. Provide an educational video and printed information

3. Expresses feelings of mastery and competence with breathing devices Rationale: School-age children strive for mastery and competence to achieve the developmental task of industry and accomplishment. Options 1 and 2 do not relate to the knowledge deficit, which is the subject of the question. Option 4 is an intervention rather than a goal

Which assessment finding following an amniotomy should be conducted first? 1. Cervical dilation 2. Bladder distention 3. Fetal heart rate pattern 4. Maternal blood pressure

3. Fetal heart rate pattern Fetal heart rate is assessed immediately after amniotomy to detect any changes that may indicate cord compression or prolapse.

An adolescent is diagnosed with iron deficiency anemia. After emphasizing the importance of consuming dietary iron, the nurse asks him to select iron-rich breakfast items from a sample menu. Which selection demonstrates knowledge of dietary iron sources? 1. Grapefruit and white toast 2. Pancakes and a banana 3. Ham and eggs 4. Bagel and cream cheese

3. Ham and eggs RATIONALE: Good sources of dietary iron include red meat, egg yolks, whole wheat breads, seafood, nuts, legumes, iron-fortified cereals, and green, leafy vegetables. Fresh fruits and milk products contain only small amounts of iron. White bread isn't a good iron source.

A newborn does not pass meconium within the first 48 hours after birth. Which diagnosis does the nurse suspect? 1. An abdominal wall defect 2. Intussusception 3. Hirschsprung's disease 4. Celiac disease

3. Hirschsprung's disease Hirschsprung's disease, or aganglionic megacolon, occurs when there is an absence of ganglionic innervation to the muscle of a segment of the bowel. Because of the absence of nerve cells, there is a lack of normal peristalsis. This results in chronic constipation. In the newborn, failure to pass meconium stools within 24 to 48 hours may be a symptom of Hirschsprung's disease.

A client arrives at birthing center in active labor. Her membranes are still intact, and the health care provider prepares to perform an amniotomy. What will the nurse relay to the client as the most likely outcome of the amniotomy? 1. less pressure on her cervix 2. decreased number of contractions 3. increased efficiency of contractions 4. the need for increased maternal blood pressure monitoring

3. Increased efficiency of contractions

A client arrives at a birthing center in active labor. Her membranes are still intact, and the health care provider prepares to perform an amniotomy. What will the nurse relay to the client as the most likely outcomes of the amniotomy? 1. Less pressure on her cervix 2. Decreased number of contractions 3. Increased efficiency of contractions 4. The need for increased maternal blood pressure monitoring

3. Increased efficiency of contractions Amniotomy can be used to induce labor when the condition of the cervix is favorable or to augment labor if the progress begins too slow.

A nurse is monitoring an infant for signs of increased intracranial pressure (ICP) and notes that the anterior fontanel bulges when the infant is sleeping. Based on this finding, which of the following is the priority nursing action? 1. Increase oral fluids. 2. Document the finding. 3. Notify the registered nurse. 4. Place the infant supine in a side-lying position.

3. Notify the registered nurse Rationale: The anterior fontanel is diamond-shaped and located on the top of the head. It should be soft and flat in a normal infant, and it normally closes by 12 to 18 months of age. A larger-than-normal fontanel may be a sign of increased ICP within the skull. Although the anterior fontanel may bulge slightly when the infant cries, bulging at rest may indicate increased ICP. Options 1 and 4 are inaccurate interventions. Although the nurse would document the finding, the first action is to report the finding to the registered nurse, who will then contact the health care provider.

A nurse reviews the record of a 1-year-old child seen in the clinic and notes that the health care provider has documented a diagnosis of celiac crisis. Which of the following symptoms would the nurse expect to note in this condition? 1. Anorexia 2. Joint pain 3. Profuse, watery diarrhea 4. Constipation

3. Profuse, watery diarrhea Rationale: Clinical manifestations associated with celiac crisis include profuse, watery diarrhea and vomiting that quickly lead to severe dehydration and metabolic acidosis. The cause of the crisis is usually infection or hidden sources of gluten. The child may require intravenous fluids to correct fluid and acid-base imbalances, albumin to treat shock, and corticosteroids to decrease severe mucosal inflammation.

When planning care for a 7-year-old boy with Down syndrome, the nurse should: 1. plan interventions at the developmental level of a 7-year-old because that is the child's age. 2. plan interventions at the developmental level of a 5-year-old because the child will have developmental delays. 3. assess the child's current developmental level and plan care accordingly. 4. direct all teaching to the parents because the child can't understand.

3. assess the child's current developmental level and plan care accordingly. RATIONALE: Nursing care should be planned at the developmental age of a child with Down syndrome, not the chronological age. Because children with Down syndrome can vary from mildly to severely mentally challenged, each child should be individually assessed. Directing all teaching to parents isn't appropriate because a child with Down syndrome is capable of learning, especially one with mild limitations.

A 3-month-old infant just had a cleft lip and palette repair. To prevent trauma to the operative site, the nurse should: 1. give the infant a pacifier to help soothe him. 2. lie the infant in the prone position. 3. place the infant's arms in soft elbow restraints. 4. avoid touching the suture line, even to clean.

3. place the infant's arms in soft elbow restraints. RATIONALE: Soft restraints from the upper arm to the wrist are appropriate because they prevent the infant from touching his lip but allow him to hold a favorite item such as a blanket. Because they could damage the operative site, such objects as pacifiers, suction catheters, and small spoons shouldn't be placed in an infant's mouth after cleft palette repair. An infant in a prone position may rub his face on the sheets and traumatize the operative site. The suture line should be cleaned gently to prevent infection, which could interfere with healing and damage the cosmetic appearance of the repair. Dried blood collecting on the suture line can widen the scar.

The nurse is preparing to care for a newborn receiving phototherapy. Which interventions are appropriate? SATA 1. Avoid stimulation. 2. Decrease fluid intake. 3. Expose all of the newborn's skin. 4. Monitor skin temperature closely. 5. Reposition the newborn every 2 hours. 6. Cover thew newborn's eyes with eye shields or patches.

4, 5, 6

The nurse is providing instructions about measures to prevent postpartum mastitis to a client who is breast-feeding her newborn. Which client statement would indicate a need for further teaching? 1. "I should breast feed every 2-3 hours." 2. "I should change the breast pads frequently." 3. "I should wash my hands well before breastfeeding." 4. "I should wash my nipples daily with soap and water."

4. "I should wash my nipples daily with soap and water." Soap is drying and could lead to cracks in the nipples.

When teaching parents of a toddler with congenital heart disease, the nurse should explain all medical treatments and emphasize which instruction? 1. "Reduce your child's caloric intake to decrease cardiac demand." 2. "Relax discipline and limit-setting to prevent crying." 3. "Make sure your child avoids contact with small children to reduce overstimulation." 4. "Try to maintain your child's usual lifestyle to promote normal development."

4. "Try to maintain your child's usual lifestyle to promote normal development." RATIONALE: The nurse should encourage the parents of a child with a congenital heart defect to treat the child normally and allow self-limited activity. Telling the parents to reduce the child's caloric intake isn't appropriate because doing so wouldn't necessarily reduce cardiac demand. Telling the parents to alter disciplinary patterns and deliberately prevent crying or interactions with other children could foster maladaptive behaviors. Contact with peers promotes normal growth and development.

A nurse is assessing a severely depressed adolescent. Which finding indicates a risk of suicide? 1. Excessive talking 2. Excessive sleepiness 3. A history of cocaine use 4. A preoccupation with death

4. A preoccupation with death RATIONALE: An adolescent who demonstrates a preoccupation with death (such as by talking frequently about death) should be considered at high risk for suicide. Although depression, excessive sleepiness, and a history of cocaine use may occur in suicidal adolescents, they also occur in adolescents who aren't suicidal. Verbal and emotional withdrawal, not excessive talking, are signs of possible depression and suicide risk in an adolescent.

A nursing student is assigned to help administer immunizations to children in a clinic. The nursing instructor asks the student about the contraindications to receiving an immunization. Immunization is contraindicated in the presence of which condition? 1. A cold 2. Otitis media 3. Mild diarrhea 4. A severe febrile illness

4. A severe febrile illness Rationale: A severe febrile illness is a reason to delay immunization, but only until the child has recovered from the acute stage of the illness. Minor illnesses such as a cold, otitis media, or mild diarrhea are not contraindications to immunization.

A nurse determines that which of the following is an appropriate short term goal for a full term, breastfeeding neonate? 1) The baby will regain birth weight by 4 weeks of age. 2) The baby will sleep through the night by weeks of age. 3) The baby will stool every 2 to 3 hours by 1 week of age 4) The baby will urinate 6 to 10 times per day by 1 week of age.

4. By 1 week of age, breastfed babies should be urinating at least 6 times in every 24-hour period.

A child is diagnosed with intussusception. The nurse collects data on the child, knowing that which of the following is a characteristic of this disorder? 1. The presence of fecal incontinence 2. Incomplete development of the anus 3. The infrequent and difficult passage of dry stools 4. Invagination of a section of the intestine into the distal bowel

4. Invagination of a section of the intestine into the distal bowel Rationale: Intussusception is an invagination of a section of the intestine into the distal bowel. It is the most common cause of bowel obstruction in children age 3 months to 6 years. Option 1 describes encopresis. Option 2 describes imperforate anus, and this disorder is diagnosed in the neonatal period. Option 3 describes constipation. Constipation can affect any child at any time, although it peaks at ages 2 to 3 years. Encopresis generally affects preschool and school-age children.

An infant is hospitalized for treatment of inorganic failure to thrive. Which nursing action is most appropriate for this child? 1. Encouraging the infant to hold a bottle 2. Keeping the infant on bed rest to conserve energy 3. Rotating caregivers to provide more stimulation 4. Maintaining a consistent, structured environment

4. Maintaining a consistent, structured environment RATIONALE: The nurse caring for an infant with inorganic failure to thrive should strive to maintain a consistent, structured environment because it reinforces a caring feeding environment. Encouraging the infant to hold a bottle would reinforce an uncaring feeding environment. The infant should receive social stimulation rather than be confined to bed rest. The number of caregivers should be minimized to promote consistency of care.

Laboratory studies are performed on a child suspected of iron deficiency anemia. The nurse reviews the laboratory results, knowing that which of the following would indicate this type of anemia? 1. An elevated hemoglobin level with a low hematocrit level 2. A decreased reticulocyte count 3. An elevated red blood cell (RBC) count 4. RBCs that are microcytic and hypo chromic

4. RBCs that are microcytic and hypo chromic Rationale: The results of a complete blood cell count in children with iron deficiency anemia will show low hemoglobin levels and microcytic and hypochromic RBCs. The reticulocyte count is usually normal or slightly elevated.

A 5-year-old child returns to the pediatric unit following a cardiac catheterization using the right femoral vein. The child has a thick elastoplast dressing. Which assessment finding requires immediate intervention? 1. One leg is slightly cooler than the other leg. 2. The leg used for the catheter insertion is slightly paler than the other leg. 3. A small amount of bright red blood is seen on the dressing. 4. The pedal pulse of the right leg isn't detectable.

4. The pedal pulse of the right leg isn't detectable. RATIONALE: Using the femoral vein during catheterization can cause the affected blood vessels to spasm or cause a blood clot to develop, altering circulation in the leg. The inability to detect the pedal pulse in the affected leg is an ominous sign and requires immediate intervention. Small amounts of coolness or pallor are normal. These findings should improve. Although the nurse should continue to monitor a dressing with a small amount of blood on it, this finding isn't the priority in this situation.

When administering an oral medication to an infant, the nurse should take which action to minimize the risk of aspiration? 1. Administering the oral medication as quickly as possible 2. Placing the medication in the infant's formula bottle 3. Keeping the infant upright with the nasal passages blocked 4. Using an oral syringe to place the medication beside the tongue.

4. Using an oral syringe to place the medication beside the tongue. RATIONALE: Using an oral syringe is the best way to prevent aspiration because it allows controlled administration of a small amount of medication. Administering the medication too quickly could cause aspiration. Putting the drug in a bottle of formula isn't preferred because the infant may not take the entire dose of medication and because the contents of the bottle could interfere with drug absorption or action. Blocking the nasal passages could cause aspiration.

Which vital sign of a newborn is abnormal 1. Axillary temperature of 37 2.RR of 50 3. BP of 70/40 4. HR of 190

4. hr of 190 -HR should be 110-160

A a nurse is reviewing care of the umbilical cord with the parent of a newborn. Which of the following instructions should the nurse include in the teaching? 1. Cover the cord with a small gauze square. 2. Trickle clean water over the cord with each diaper change. 3. apply hydrogen peroxide to the cord twice a day. 4. Keep the diaper folded below the cord.

4. keep the diaper folded below the cord -the cord should not be covered because it promotes infection -water should not be applied to cord -cord should be kept clean and dry. hydrogen peroxide not applied to cord

A student is assisting the healthcare provider to care for a baby diagnosed with Down syndrome (DS). Which the following statements made by the student indicates the student requires additional instruction about the disorder? A) "Both male and female patients diagnosed with Down syndrome are infertile." B) "The baby will be scheduled for an ultrasound of the heart to check for problems." C) "The baby's red and white blood cells and platelets will need to be monitored closely." D) "We will be checking the baby's thyroid hormone levels now and periodically."

A) "Both male and female patients diagnosed with Down syndrome are infertile."

A patient who is 25 weeks pregnant has partial placenta previa. As the nurse you're educating the patient about the condition and self-care. Which statement by the patient requires you to re-educate the patient?* A. "I will avoid sexual intercourse and douching throughout the rest of the pregnancy." B. "I may start to experience dark red bleeding with pain." C. "I will have another ultrasound at 32 weeks to re-assess the placenta's location." D. "My uterus should be soft and non-tender."

B. All the other options are CORRECT about partial placenta previa. Option B is WRONG because this condition will present with PAINLESS, bright red bleeding NOT with pain and dark red bleeding, which happens in abruptio placentae.

The healthcare provider is assessing a child who has a diagnosis of autistic spectrum disorder (ASD). Which of these clinical findings supports this diagnosis? A) Annoys others deliberately B) Uninterested in playing with others C) Cries for attention at inappropriate times D) Utilizes manipulative behavior

B) Uninterested in playing with others

A child is diagnosed with autistic spectrum disorder (ASD). Which of the following, if present in the patient's health history, will the healthcare provider identify as a factor associated with this disorder? Choose all answers that apply: A) Southeast Asian or Middle Eastern descent B) Sibling diagnosed with Asperger syndrome C) Concurrent diagnosis of fetal alcohol syndrome D) Advanced age of the mother or father E) Exposure to vaccines containing thimerosal

B, C, D

A 28 year old female, who is 33 weeks pregnant with her second child, has uncontrolled hypertension. What risk factor below found in the patient's health history places her at risk for abruptio placentae?* A. childhood polio B. preeclampisa C. c-section D. her age

B. Preeclampisa is a risk factor for experiencing abruptio placentae. The patient is at risk for developing this condition again since she is currently experiencing uncontrolled hypertension with this pregnancy.

A nurse reviews the record of a child who was just seen by a health care provider (HCP). The HCP has documented a diagnosis of suspected aortic stenosis. Which clinical manifestation that is specifically found in children with this disorder should the nurse anticipate? A. Pallor B. Hyperactivity C. Exercise intolerance D. Gastrointestinal disturbances

C. Exercise intolerance Rationale: The child with aortic stenosis shows signs of exercise intolerance, chest pain, and dizziness when standing for long periods. Pallor may be noted, but it is not specific to this type of disorder alone. Options 2 and 4 are not related to this disorder.

You're assessing the five minute APGAR score of a newborn baby. On assessment, you note the following about your newborn patient: heart rate 97, no response to stimulation, flaccid, absent respirations, cyanotic throughout. What is your patient's APGAR score?* A. APGAR 2 B. APGAR 3 C. APGAR 0 D. APGAR 1

D. APGAR 1:...A: 0, P: 1, G, 0, A: 0, R: 0

Stage 1 of labor includes which phases in the correct order?* A. Transition, Latent, Active B. Active, Latent, Transition C. Active, Transition, Latent D. Latent, Active, Transition

D: Latent (early labor), Active, Transition

Which signs and symptoms characterize cold stress? Select all that apply. Fever Tachycardia Mottling of the skin Periods of apnea Hyperactivity

Mottling of the skin Periods of apnea Signs and symptoms of cold stress include decreased skin temperature, increased respiratory rate with periods of apnea, bradycardia, mottling of skin, and lethargy.

Disseminated intravascular coagulation (DIC) can occur in __________________. This happens because when the placenta becomes damaged and detaches from the uterine wall, large amounts of _____________ are released into mom's circulation, leading to clot formation and then clotting factor depletion.* A. Placenta previa, fibrinogen B. Placenta previa, platelets C. Abruptio placentae, fibrinogen D. Abruptio placentae, thromboplastin

The answer is D.

A a nurse is caring for a newborn immediately following a circumcision using a gomco procedure. Which of the following actions should the nurse implement? a. apply gelfoam powder to the site. B. Place the newborn in the prone position. C. apply petroleum gauze to the site. d. scrub yellow discharge

c. apply petroleum gauze to site -prevents adhesion to diaper


Conjuntos de estudio relacionados

A&P II Chapter 24 Mastering A&P Practice Metabolism

View Set